Anda di halaman 1dari 84

Acquisition of Property

I.
Scope of coverage We talked about different ways of acquiring property: A. Discovery B. Capture C. Finding D. Adverse possession Acquisition of property by discovery A. Doctrine of first possession property belongs to discoverer. B. Departure from the doctrine of first possession to suit the needs of conqueror: Johnson v. MIntosh (page 3, Johnson obtained title from Indian Tribe chief, MIntosh obtained title from US government. Indian Tribes did not have title to land in America despite first possession. The conqueror can set the rules)

II.

III. Acquisition of property by capture Applies to wild animals and natural resources

A. General principle: the first capturer gets the property. Pierson v. Post (page 17, fox hunter
v. fox killer). 1. Note: if the first capturer is trespasser, the land owner prevails.

B. In absence of actual physical possession, consider the following factors: 1. Trapped or fatally wounded or deprived of its natural liberty? If yes, there is 2. 3. 4.
constructive possession. This is a better standard than the dissent because it is easier to make a conclusion on. Reasonable prospect of capturing the wild animal? If yes, some courts will find constructive possession exist (Dissent in Pierson v. Post, page 17) Another standard is all that was possible (Ghen v. Rich) Some courts will adhere to trade custom (example: Ghen v. Rich, page 23, whale killer v. whale finder). In determining whether to follow the trade custom, courts look at: a. Whether the custom is universally followed in the industry or if it is regional; b. Whether the custom affects only people in the trade; c. Whether there are competing customs involved in the case; d. Whether the underlying activity is a trade or sport. Whether the possession process was interrupted by malicious intervention of the defendant. If yes, there is an argument for unfair competition. However, no cause of action exists if defendant was using lawful means of competition (example: Keeble v. Hickeringill, page 27, decoy duck pond and defendants firing of shot guns). Must be lawful trade + malicious intervention Whether possession was interrupted by wrongful conduct of a 3rd party. If yes, argue for pre-possessive interest (example: Popov v. Hayashi, page 110, baseball).

5.

6.

C. Escaped animals 1. Return to natural habitat a common property again. This is a legal standard 2. Tenancy to come back - animals that exhibit tenancy to return has not returned to natural habitat and thus owner does not lose right (example: Note 2, page 32, two foxes imported from Canada, the male fox escaped and killed a short distance away). Or the example of deer from our book on 33

D. Natural resources Rule of first possession applies. Example: There is oil in the underground of A and B. B drilled a well on his property and started extracting the oil. Can A sue to stop B from doing this? Answer: courts generally hold that the natural resources are like wild animals and thus the rule of first possession prevails. 1. Re-injected natural resource not deemed to have returned to natural habitat. IV. Acquisition of property by finding A. General principle: Finders title is good against everyone except true owner, prior finders, and sometimes land owner. This is based on the doctrine of first possession.

1. Example: Armory v. Delamirie (page 96, chimney sweeper boy v. goldsmith for a

jewel. Chimney boy prevails). 2. Example: Anderson v. Gouldberg, (Note 4, page 97, the plaintiff trespassed on a timberland of a 3rd party, cut logs, and hauled them to a mill, where the defendant took them. Plaintiff sued to repossess the logs. The court ruled for the plaintiff). 3. Example: A stole a watch from O, B stole the watch from A. In a dispute between A and B, A prevails. In first possessor v. subsequent possessor who is not true owner, how first possessor obtained possession is irrelevant. Rationale: to discourage a chain of wrongdoing. 4. What constitutes possession by finder? Example: Oklahoma cash (Note 3, page 106). B. Finder v. landowner Different courts have focused on different factors which may lead to different results: 1. Factor 1: The position of the property If the found property is attached to the land or underneath the land, courts are more likely to find that the title goes to the landowner who is deemed to have constructive possession of everything that is attached to or underneath his land. Example: Staffordshire Water Co. v. Sharman (cited in Hannah v. Peel on page 102, rings embedded in the mud at the bottom of the swimming pool the defendant was hired to clean). Contrast Bridges v. Hawkesworth (cited in Hannah v. Peel on page 100, British government notes lying loose on the shop floor). 2. Factor 2: Owners possession of the land The owner of the land is deemed to have constructive possession of the found property by virtue of his possession of the land. However, if the owner never took possession of the property, courts are more reluctant to find in favor of the landowner. Example: Hannah v. Peel (page 99, broach found in house which true owner never occupied). 3. Factor 3: the relation between finder and landowner a. If finder is an employee or a hired worker, then court may find that the finder is an agent of owner and give title to owner. Some courts have looked a step further at whether the finder is under any contractual obligation to report (example: Note 4, page 107, two laborers hired to build driveway but dug out a jar of gold coins. Court found in for landowner because the two laborers were his agents). b. Trespassing finder loses to landowner as a result of strong policy against trespass (example, Note 3, page 104, Peel found the broach while trespassing on Hannahs property).

4. Factor 4: whether the property is abandoned, lost or mislaid a. If property is abandoned, then first finder gets it unless he is deemed an agent of the landowner. b. If the property is misplaced (i.e., the true owner intends to retrieve it), the court will likely give title to the landowner, because he has a better chance of returning the property to the true owner. Example: McAvoy v. Medina (page 105, wallet found lying on counter of barber shop). c. If the property is lost, there have been cases in which the court held that the finder gets title unless he is deemed agent of the landowner (example: Bridges v. Hawkesworth, cited in Hannah v. Peel on page 100, British government notes lying on shop floor). C. Finder of abandoned property 1. General rule: Finder gets title. 2. Intent to abandon is key: finders reasonable belief under the circumstances.

3. Example: A was moving into an apartment in New York City. A unloaded his truck and

his personally belongings were scattered around the entrance of the apartment building and on the side walk. A few paintings were actually close to a public rubbish bin. While A was moving his stuff into his apartment, B was passing by and saw the paintings. He took the paintings to a nearby gallery and said: These paintings were abandoned but I think they may still be of some value. The owner of the gallery inquired about the circumstances in which he found the paintings and afterwards took the painting and paid B $200. Upon discovery, A sues the gallery for the return of the paintings. Question: Can the gallery argue that it was a good faith purchaser of an abandoned property? Answer: Likely no, because a reasonably person would have guesses that someone was in the process of moving into the apartment and the paintings were placed there to be taken inside later. Discovery Group v. Atlantic Mutual Insurance Co., 974 F. 2d 450 (4th Circuit, 1992). In 1857 the steamship Central America sank in a storm off Cape Hatteras, carrying large cargo of California gold to the bottom. The insurers paid off, and thus acquired ownership of the insured cargo. In 1988 the wreck of the Central America was discovered and part of the cargo recovered. Can the insurance company that paid off years ago claim title to the recovered cargo? Answer: Though 131 years had elapsed the 4th Circuit ruled that the mere elapse of time was not enough to prove abandonment.

4. Mere lapse of time is not enough to show abandonment (example: Columbus-America

5. There is an objective standard

a. A reasonable person under similar circumstances would believe that the


property is abandoned. Note that this can be muddled between finding and taking possession of an item. If you see a wallet and tell your friend about it and he picks it up V. Acquisition of property by adverse possession A. Rationale for awarding adverse possession 1. Sleeping theory

a. Encourage maintenance and upkeep of the property b. A person who doesnt claim their property can be assumed as abandonment c. the law gives a statute of limitations for people to come forward during a specified period of time or else they lose title. It is used to punish the lazy owner. 2. Earning theory a. The adverse possessor took the time and effort to live there, make improvements, and make it his own. B. Elements of adverse possession Actual and exclusive, adverse and under claim of title, open and notorious, continuous. 1. Element 1 - Actual and exclusive possession a. Actual possession - highly factual and likely an issue of disagreement (example: Van Valkenburgh v. Lutz, page 115, majority and dissent differed on whether Lutzs partial occupation of the disputed land satisfied the actual possession requirement). The key is whether the possession is akin to that of a true owner.

b. Exclusive possession: possession must be to the exclusion of others as a


reasonable true owner would.

i.

Example: A purchased a hunting ground from O under a deed that was defective. He built a cabin and put a Private Property, No Trespassing sign at the entrance of a footpath leading to the cabin. Occasionally some nosy people got close to the cabin and looked inside. A ignored those people most of the time and occasionally engaged in friendly conversations with them. Upon realizing that the deed was defective, O claimed title in himself but A claimed adverse possession. Was As possession exclusive? Answer: Yes, it was used in a way that a true owner would have used the property.

2. Element 2 Adverse use under claim of title a. Possession must be without permission of true owner. b. When property is co-owned, adverse use can be established only when one coowner excludes the other co-owner from possession or acted in a way that is inconsistent with the co-ownership interest of the other co-owner. c. What constitutes under claim of title? 3 different views. i. Objective view (most US jurisdictions): subjective mental state of adverse user is irrelevant. Instead, focus on whether the adverse possession objectively appears to be under claims of ownership. 1. Whether your possession is under the impression of a reasonable true owner 2. And whether you didnt have permission to be there in the first place. ii. Maine Rule: I knew I did not own it, but I occupied it anyway. 1. Van Valkenburgh case 2. Encourage malicious wrongdoing in the first place. iii. Good faith belief: I honestly thought I owned it. 1. Your mental state should be such that you truly owned this place. 2. Ask her whats wrong with this position.

d. Disclaimer by adverse possessor of title stops the adverse use and the running

of the statute of limitation. Example: Squatter occupies Blackacre and satisfies all the elements of adverse possession for 9 years. The limitations period is 10 years. Then Owner discovers Squatters presence and confronts him. Squatter says, Ill leave as soon as I can arrange a new place. Just dont sue me. If Squatter stays for another 2 years, can he acquire title by adverse possession? No. He disclaimed ownership. Adverse claim of right element has been missing since then. requirement.

e. Possession under color of title (defective deed) satisfies adverse claim of title

f. Permission i. Silence does not equal permission because this goes to the underlying rationale of adverse possession itself because adverse possession is meant to punish slothful owners who sit on their rights, therefore, if silence is not permission, then it furthers the purposes of adverse possession doctrine ii. Family members 1. Courts are split on this issue and some presume permission when family is involved and others do not iii. Co-owners 1. A course of action must first be triggered in order to make the statute of limitations begin to run 2. A course of action must be a violation of the other coowners rights to the property 3. One co-owner must be ousted by the other owner so as to make his possession adverse against the interest of the other co-owners 4. Key Issue: Ouster Test g. Defective deed i. Hypo: O conveyed deed to A. A began possession of the property throughout the limitation period but O finds that the deed was defective and now comes forward and tells A to leave. ii. Technically As possession was under permission of O but to deprive A of the title to the land in such circumstances would be harsh and therefore the law says: iii. If A enters the land under defect of deed, the law regards that A has satisfied the requirement for under adverse claim of title (even though technically A had permission) 3. Element 3 Open and notorious a. Readily visible upon reasonable inspection. b. True owner standard: open and notorious satisfied if adverse possessor acted in a way that the true owner would have.

c. Actual knowledge by true owner starts the limitation period even though adverse
possession is not open and notorious. Rationale: actual notice is better than constructive notice. If you do not know your rights as an owner are being infringed you shouldnt be punished.

d. Underground occupation actual notice by true owner is required (example:


Marengo Cave Co. v. Ross, Note 3, Page 125, a cave lies underneath property of

A and B with entrance on As land. A opened the cave to the public for a fee during the statutory period).

e. Boundary disputes: Encroachment must be self-evidently apparent to naked

eyes upon reasonable inspection. A few inches of encroachment is not open and notorious. Actual knowledge is required (example: Mannillo v. Gorski, page 130, 15 inches of encroachment on neighbors property does not satisfy open and notorious requirement). i. What constitutes actual knowledge? See Note 2, page 134 (I hear there is a problem with your neighbors house the new part they built may be too far. The answer depends on how narrowly the court interprets the notice requirement. Some require actual knowledge, others require the true owner to take reasonable steps to investigate if thats what a reasonable person would have done). ii. Knowledge of what? Note 2, page 134. Knowledge of smaller encroachment than the actual encroachment not sufficient for actual knowledge. iii. Equitable solution in lieu of injunction if undue hardship on the part of adverse possessor with good faith belief in his title: pay for the land claimed. Alternatively, the true owner may be required to pay for the improvement.

f. If the adverse possessor possesses the true owners property secretly, it is difficult for the true owner to take notice of the violation that someone else has done to his property g. The law says the adverse possessor must have open and notorious possession such that the true owner of the property was given adequate notice h. Open and Notorious: i. Reasonable person would be put on notice OR ii. Possession as if true owner would i. HYPO: a family is living on a tract of land owned by a company used for timber and oil. Is the familys occupation open and notorious? i. This depends on the factors j. HYPO: A goes to his neighbors yard at 2 am every morning so he doesnt satisfy the open and notorious requirement, however, the true owner was aware of his secret act and had actual notice of his possession. Would you grant title to A under adverse possession? i. Even though the open and notorious notice is missing in this claim, A can still prevail because the true owner has actual notice that his interests are being violated ii. Open and notorious requirement is intended to provide the true owner constructive notice that his rights have been violated and actual notice is even better than constructive notice so in absence of constructive notice iii. If you can show the true owner had actual notice, it satisfies the rationale for granting title under adverse possession k. What constitutes actual notice? i. HYPO: Your neighbor is building a shed, supposedly on his yard and you believed it to be in his yard. Another neighbor tells you that he heard that the other neighbors shed may have gone too far and that the true owner should check it out. The true owner does nothing but the shed did encroach upon his property. Was the true owner given actual notice? 1. The law says that the true owner has a duty to investigate only if a reasonable person under similar circumstances would have investigated

l.

If it is an 85 year old we still go by the average property owner standard

4. Element 4 continuous possession a. Statutes of limitation stops running if possession is interrupted. i. Rationale:

b. Constant possession is not required, focus on the true owner standard

(example: Howard v. Kunto, page 136, summer house used during summer time only satisfied continuous possession). i. It is reasonable to believe that a true owner would only use this land during the summer months as well.

c. Tacking:

i.

Privity (voluntary conveyance) required and satisfied by showing voluntary conveyance between adverse possessors of deed or possession. Conveyance by a defective deed is indication of voluntary conveyance of possession. Rationale for privity requirement: discourage acquisition of property by unlawful means. 1. Punish the true owner. He has been sitting on his property rights for a long time. 2. Even though there is not value paid by the current possession, the mere fact that it was voluntarily transferred means that he voluntarily gave up all of his rights to this piece of property. 3. It can be a transfer of possession or a transfer of a deed. 1. Even if the deed was defective, if there was a clear intention of transfer of possession then it is ok. 4. Policy reason: 1. We dont want to encourage criminal acts. Used to deter wrongdoing and trespassers. Example: A had been an adverse possessor for 19 years and 11 months and spent the following month in Ireland for a family reunion. B, upon learning the nature of As possession and while A was away, took possession of the house. Can B tack on As 19 years of possession for purpose of adverse possession against the true owner? Answer: No. To hold otherwise would encourage trespassing and unlawful taking of property. was ousted by B at gunpoint. B was not the true owner. A had all the intention of coming back to the property when circumstances would permit. 3 years later, while B was still in possession of the property, the true owner showed up and B fled. The statute of limitation is 10 years. Can A still claim adverse possession? Answer: prevailing view in the US says the statute of limitation is suspended. Is this view consistent with the rationale for awarding adverse possession? No, if the rationale is sleeping theory; but Yes if the rationale is earning theory (since A has not paid enough dues by occupying property for the required amount of time. 1. Can A tack on Bs possession to claim adverse possession against owner? 1. B cant because it wasnt voluntarily given to him.

ii.

iii. Example: Suppose A had been occupying the premises for 8 years when he

2. Punishing theory: it doesnt matter who was possessing the land; the owner was slow in claiming his rights 3. Rewarding theory: A, regardless of the reason, has been on this property for only 8 years. Its too bad he was ousted, but he didnt meet the statute of limitations. C. Effects of oral relinquishment Once title is established by adverse possession, it is not lost by divestment or oral relinquishment (example: Note 3, page 134, A and B own adjacent lots. A erects a fence on what she mistakenly believes to be the true boundary line. In fact the fence is on Bs lot 3 feet beyond. A thereafter lived there throughout the entire statutory period. After the statute has run, a survey by B revealed the mistake. A, to avoid a hassel, tears down her fence and erects a new fence on the original true boundary. 3 years later, A talks to a lawyer, changes her mind, and sues to eject B from the 3 feet. What result? Answer: A wins, absent some sort of estoppel arguments (e.g., that might arise if B subsequently improved the land) because she had already obtained title by adverse possession when she torn down the fence). D. Title acquired by adverse possessor 1. Gets only what the true owner has. 2. Example: Note 3, Page 142: O, owner of Blackacre, dies in 1991 leaving a will that devises Blackacre to B for life, remainder to C. In 1992 A enters adversely upon the land. In 2006 B dies. Who owns Blackacre? Answer: C, because A can only acquire the interest of B, which is a life estate. B was in legal possession of the property when adverse possession happened. The nature of Bs interest lasts only during his life. Yes, adverse possessor prevails over B, but keep in mind he gets no more than what B is entitled to. Bs interest ends when he dies in 2006. E. Extent of title acquired by adverse possession 1. Entry without color of title: only the area actually possessed.

2. Entry with color of title (defective deed): all land described in the defective deed if it

is a single parcel of land and a significant portion of the parcel occupied. Constructive possession exists for the unoccupied portion.

3. Constructive possession yields to actual possession (example: A granted a deed to B for 100 acres of land. The deed turned out to be defective. A has been occupying 50 acres of the land, but unbeknown to B, C has been occupying 10 acres of the remaining 50 acres of land throughout the limitation period. B can claim title to only 90 acres of land. Bs constructive possession with regard to the 10 acres occupied by C is inferior to Cs actual possession).

4. If adverse possessor realizes he occupied land that was not his and abandons

property and tries to reclaim. Once he has acquired adverse possession then the property is his. There must be some sort of transaction that the original adverse possessor must convey title to give up his interest.

F. Suspension of statute of limitation due to disability of true owner 1. Common types of disability: insanity, imprisonment, minor

2. Be careful: statutes in many jurisdictions required disability exists at the time the cause of action accrues, i.e., the start of adverse possession.

3. Example: A started adverse possession in 2000. O was the true owner of sound mind.

In 2001, O died and left the land to his 2-year old nephew. Assume statute of limitation period is 10 years. When does the limitation period end? Answer: 2010. The nephew will be 12 years old. His guardian should bring lawsuit on his behalf.

G. Adverse possession of personal property 1. Typically shorter limitation period (e.g., 6 years)

2. Focus on the conduct of true owner upon discovery of loss: cause of action will not

accrue until the injured party discovers, or by exercise of reasonable diligence and intelligence should have discovered, facts (including the identity of the holder of the property) which form the basis of a cause of action. Burden of proof is on true owner (example: OKeeffe v. Snyder, page 144, case remanded for further findings along this guideline).

3. In NY, statute of limitation does not begin to run until true owner demands return and is refused.

Landlord and Tenant


I. Scope of coverage A. Types of lease B. Law on selection of tenants C. Assignment v. sublease D. Landlords obligations and tenants remedies E. Tenants obligations and landlords remedies Types of lease A. Four Types 1. Term lease 2. Periodic Lease 3. Tenancy at will 4. Tenancy at sufferance B. Term of years 1. A lease for a single fixed term of any length.

II.

2.

Beginning and ending dates must be either set out in the lease or determined by reference to a formula. Example: from August 1, 2007 to the end of the school year of UC law school in 2010. Lease of indeterminable duration: for as long as the war. Common law treats this type of lease as tenancy at will. Some courts regard it as term of year lease since the parties intend the lease to last at until the war ends. No notice of termination is required at the end of the lease term.

3.

4.

C. Periodic Tenancy

1.

A periodic tenancy is a lease for a period of some fixed duration that continues for succeeding periods until either the landlord or the tenant gives notice of termination (example: to T from year to year. If notice is not given, the period is automatically extended for another year). a. You cant terminate the lease within the period. You have to terminate the lease at the end of the period. The notice requirement a. Annual lease: 6 month notice. i. A lease starts on July 1, 2009. You want to get out of your lease. You give notice on Jan 4, 2010 that you will no longer be there. (1) Not acceptable. The last day of your lease is June 30 2010. You need to give notice BEFORE midnight on December 31, 2009 in order to terminate your lease. (6 months notice before the end of the lease) b. Periodic lease of less than 12 mos: notice time = length of lease period, not more than 6 mos. Under the common law, the termination day must be the final day of the lease, not in the middle of it. However, some states may alter that by statute (example: Month-to-month lease starting from January 1, 2008. On April 3, T gives notice to terminate. What is the earliest date of termination? Answer: May 31, 2008, not April 30 or May 3). i. Lease of 9 months Jan 1, 2008. Tenant gives notice on November 15, 2008. What is the earliest termination date of this lease? Answer: June 30th, 2009 is the last day of the lease. Notification needs to be given 6 months before this so the lease can end on this date. . (1) First figure out the last day of the current period. From there, we backtrack to provide the notice. That would be the last day on which notice of termination can be given. Effect of death of landlord: The death of the landlord of tenant has NO effect on a periodic lease. However, his estate may elect to terminate the lease at the END of the lease period by giving the required notice. EXAMPLE: a. To A from year to year.

2.

3.

4.

D. Tenancy at will 1. Definition: no fixed term and lasts as long as both landlord and tenant wish to continue. a. L leases land for T for and during the pleasure of the landlord-L and T can terminate it at any time.

2.

Right to terminate: a. Either party may terminate. b. If the lease provides that it can be terminated by one party, common law will give the same right to the other party. What if the lease has given the right to terminate this lease to only one party?

3.

a. Leases with unilateral right to terminate may be deemed as something other than a
tenancy at will (example: Garner v. Gerrish (page 305 ) The lease said it shall continue for and during the term of quiet enjoyment from the first day of May, 1977, but T has the privilege of termination of this agreement at a date of his own choice. It was a determinable life estate).

10

b. If it is a tenancy at will, it means that either party can terminate it at will. If the lease has given right to only one party, the law will grant the same right to the other party=common law i. This sort of treatment is inconsistent with the intention of the parties. (1) For this reason, in leases that have granted unilateral rights to terminate to one party, courts are more inclined to treat this lease as something other than tenancy at will. (2) Courts recognize that if we treated it as a tenancy at will, wed have to grant the same rights to the other party. (3) They try their best to interpret the lease so the lease can be deemed something other than a tenancy at will. c. L leases land to T so long as T should wish i. This creates a determinable life estate because only T can terminate it. ii. L has a reversion

4. 5. 6.

Effect of death of one party: ends the lease. Notice of termination ahead of time: No requirement of advanced notice under common law, but state statues may require a period of notice. What if it as many years as the landlord desires? Tenancy at will. But modern courts would still regard it as a life estate in tenant. It is a determinable life estate because it depends on the wish of the landlord. a. Another way the modern courts avoid an undesirable outcome.

E. Tenancy at sufferance Tenant holds over after termination of the tenancy. At the end of the lease, the tenants refuse to leave. It is wrongful for T to holdover on this land after the termination of the lease. It is terminated when the landlord evicts the tenant or chooses to hold the tenant over to another term. III. The law on selection of tenant A. Applicable law 1. Common law: free to discriminate.

2.

Civil Rights Act of 1866: All citizens of the United States shall have the same right, in every state and territory, as is enjoyed by white citizens thereof to inherit, purchase, lease, sell, hold, and convey real and personal property Federal Fair Housing Act: prohibits discrimination based on race, color, religion, sex, familiar status, national origin. Prohibited activities include refusal to rent or sell, to discriminate in the terms, conditions of the lease or sale, to say property is not available for lease or sale when in fact it is available, to put inappropriate language in advertisements, to induce or attempt to induce any person to sell or rent any dwelling by representation regarding the entry or prospective entry into the neighborhood of persons of a particular race, color, etc.

3.

a. Exemption: single-family house sold or rented by an owner: provided, such private


individual owner does not own more than 3 such single-family houses at any one time, etc.

11

4.

Burden of proof: plaintiff must show discriminative effect, and defendant has burden of showing lack of discriminative motive (i.e., there is legitimate reason for the discrimination). Example: Note 3, page 380, Murphy has an apartment to rent in her home. She puts the following advertisement in a local newspaper: For rent, Furnished basement apartment in private white home. Call . A couple of African American origin applied and is rejected by Murphy because of race. Has this violated law? Answer: her advertisement violated FHA, her refusal to rent violated Civil Rights Act of 1866 but not FHA (due to single-family exemption).

5.

a. Example: Note 3, page 380, third full paragraph: Does the regular exclusion of

minority models from real estate advertisements containing human models violate the FHA? Answer: Court said FHA prohibits all ads that indicate a racial preference to an ordinary reader whatever the advertisers intent. If an ad that includes a single model or a couple of one race that is run only two or three times, it is unlikely to be found in violation of FHA. However, if there is a long-standing pattern of publishing real estate ads in which black models are used for housing in predominantly black neighborhoods and white models are used for housing in predominantly white neighborhoods, a case may be established under FHA. houses for rent. They limit the number of occupants in each house. One of such houses is a 1,200 s.f house, consisting of 1 living room, two bedrooms, two baths, a den opening directly into the living room (which could be used as a bedroom). They limit the number of occupants to 4 people and refused to rent it to a couple with 3 children. Does this violate FHA because it discriminates based on family status? Answer: No, the landlords could establish legitimate business reason to prevent over usage of the apartment beyond its normal wear and tear.

b. Example: Note 4(a) Page 381, landlords, a retired couple, have 8 single-family

c. Example, Note 4(d) Page 381: L refuses to rent to a gay couple because he
objects to the partners sexual orientation. Answer: the FHA prohibits discrimination based on sex, but does not prohibit discrimination based on sexual preference.

d. Example discrimination based on handicap1: L refuses to rent to T who is


known to L as a drug addict. Can T argue that his addiction to drug is a handicap and thus L violated FHA? Answer: No, because FHA handicap definition does not include illegal drug addiction.

1. What about AIDS? Yes it is considered a handicap and the landlord cant
refuse to rent just because the tenant has AIDS.

e. Example: Note 4(g), page 382: L wants to evict T because T has a mental
disability that results in seemingly threatening behavior. Question: Is mental illness a handicap under FHA? Answer: Yes. However, court has held that L is not obligated to rent to T if his condition would create a direct threat to the health and safety of others or result in substantial damage to the property, but L must make reasonable accommodations. Interpretation: L must at least give T reasonably amount of time to locate another place and move out.
1

Definition of handicap under FHAL it means a physical or mental impairment which substantially limits one or more of the persons major life activities.

12

1. If keeping the tenant there would potentially harm others or the


property, the landlord if not required to keep this tenant. This is something beyond reasonableness.

f. Example: landlord has a no pet policy but a tenant needs the dog because she is schizophrenic and relies on the dog as a friend. Can the landlord discriminate and not let her have the dog? 1. The landlord needs to accommodate to allow the person to keep the dog as long as she can prove that the dog is a necessary part of treatment and that there is no other reasonable way to accommodate this disability. g. A landlord can discriminate based on profession. Some state statutes have a law prohibiting the landlord from discriminating on profession, but not all. IV. Assignment and sublease A. B. C. D. Assignment Privity of Estate- mutual or successive relationship to the same rights of property. or Privity of Contract Subrogation Theory- assuming you do not have the standing to sue someone do you have the standing to sue a third party or can they sue you 1. Definition: L or T has transferred his entire interest in the lease. 2. After assignment, who has standing to sue if either party fails to perform lease obligations? Privity of contract or privity of lease must exist between plaintiff and defendant.
What constitutes release? Must be explicit. L Privity of contract unless release T Privity of contract
n by T1

Pr i vi ty

Pr ivi ty of of co es nt tat ra e ct if a ss um pt io

T1 What constitutes assumption? -Not implicit in assignment; -subject to lease or subject to obligations and covenants in lease not sufficient

3.

Example, problem 2(c), page 394: L -- T T1 (assumption of all obligations) T2 T3. T3 defaults on rent payments and fails to keep premises in good shape. L sues T, T1, T2 and T3. What are their liabilities to L and among themselves? Answer: a. T to L: T is liable to L for T3s default under privity of contract. There is no novation (i.e., release by L of Ts obligation) of the original lease and its covenants. b. T1 to L: T1 is liable to L for T3s default on privity of contract, by virtue of T1s assumption of the covenants in the lease. c. T2 to L: Not liable to L. T2s privity of estate with L ended when he assigned the lease to T3, and T2 was never in privity of contract with L because he did not assume the covenants of the ORIGINAL lease. (the lease between L and T; it isnt sufficient to explicitly assume the obligations in the contract between T1 and T2)

13

d. T3 to L: T3 is liable to L on privity of estate arising from the assignment. One in

privity of estate with L is liable for his breaches of all covenants in the lease that run with the land. e. T against T1: If L proceeds against T, T can proceed against T1 on privity of contract (because T1 assumed the covenant in the lease). In addition, T can proceed against T1 and/or T3 on subrogation theory T stands in the position of L. f. T against T2: No privity of estate, no privity of contract, and no subrogation (since L has no cause of action against T2). Thus, there is no cause of action. g. T against T3: No privity of contract, no privity of estate, but if L recovers from T, T can go after T3 by standing in the shoes of L (subrogation theory). h. T1 against T3: Similarly, if T proceeds against T1, T1 may proceed against T3 on the same subrogation basis (although there is no privity of contract nor privity of estate between them). E. Sublease 1. Definition: lessee transfers anything less than his entire interest in the leasehold, thereby retaining a reversion.

2.

The standing to sue:


L
Pr iv it y

Privity of contract Privity of estate

ra

ct

if a

ss um

pt

io

by

T1

Privity of estate T1

co nt

3.

Note: although L cannot sue T1 for damages in absence of T1s explicit assumption of lease, L can terminate the lease and evict T1.

F. Distinguishing assignment from sublease 1. Traditional common law (also majority of US jurisdictions): sublease has shorter term than original lease. a. Full remaining period? i. Whether or not T has retained any right to re-enter the property. ii. If the transfer is for the entire remaining term of the lease, it looks like an assignment, but we still need to look if the transferor has retained a right to reenter the property (1) T, in the transfer agreement, says on the last day of this transfer/lease, I have the right to come to the property and inspect the property. (2) If T1 defaults on any of his obligations, I have the right to re-claim the property. (3) If transfer agreement says at the end of this lease term, T has the obligation to restore this leased premise to its original state.

2.

Significant minority: look at whether the parties intend to create a reversion in the original tenant. Mere use of words of sublease or assignment in the document is not

14

Privity of contract

of

conclusive evidence of intent (example, Ernst v. Conditt, page 388, court held the transfer was an assignment despite use of term sublease repeatedly in document because T did not reserve any right to enter premises at the end of the lease term, did not reserve the right to re-enter upon breach of lease term by T1, and T1 was to restore the property to original state at the end of the lease term). a. Whether or not the transferor can re-possess the property if T1 defaults, whether T has the opportunity to restore the property to original estate, whether T has the right to re-enter the property to make sure it is in good shape on the last day of the lease. b. Not that much different from the majority view. G. Restrictions on assignment and sublease 1. General rule: unless the original lease explicitly prohibits assignment or sublease, lease is freely transferable by either method. Any restrictions are narrowly construed. a. If the lease says T cant assign this property to any other person. T turns around and subleases it to another person. L objects, but the law says that the original restriction only prevents T from ASSIGNING the lease, not subletting. b. The law wants to promote more productive use of the property. c. Very often we will see a provision that says tenant cant transfer the interest in the lease without the landlords consent. i. Does L have absolute power to withhold consent???? d. Can transfer an explicitly denied lease even with death if it is to family member, because it does not say transfer by inheritance. See policy reason in b 2. Limits on landlords power to withhold consent of transfer: a. Common law: landlord may withhold consent even without good reason if the lease so provides.

b. Modern view (increasing minority): landlord cannot withhold consent without


reasonable commercial objections, unless the lease explicitly provides that the landlord may withhold consent with or without cause. There is a growing view that a lease is a contract in nature and as such, there is implicit requirement of good faith and fair dealing (example, Kendall v. Ernest Pestana, Inc., page 395)

i.

When the parties dont have equal bargaining power, the court should impose a reasonableness requirement even when the lease explicitly says T cant transfer without Ls consent. L has the absolute discretion on this consent. Landlord can insert into contract landlord reserves right to withhold consent to sublease/assignment with or without reason or original tenant remains liable

ii.

c. What is a commercially reasonable objection? i. ii. iii. Economic rather than personal concerns. New tenants ability to pay rent. Alteration to the existing premises (1) The premise is for office space, but the new tenant wants to come in and open a restaurant. iv. General economic interest of the landlord?

15

(1) Landlord argues that he doesnt want to give consent because he can push
him into paying a higher rent. This is not a commercially legitimate reason v. Unreasonable (1) Denial without stating reasons (2) Failure to respond within a reasonable period of time (3) Personal taste or demeanor (e.g. morally object to an abortion clinic) V. Landlords obligations and tenants remedies A. Landlords obligation to deliver legal possession 1. Common law: landlord is required to deliver only legal possession, i.e., no one claiming through landlord has a better right to possession than the tenant. a. Legal possession: no one holds any superior right to possess the leased property during the leased term than the tenant. b. True owners have superior legal right to possess the property. This is a classic example of a landlord failing to deliver legal possession. He is REQUIRED to maintain tenants superior legal right throughout the entire lease period. 2. It is a continuous obligation on the part of the landlord throughout the lease term. Tenant may waive his right to receive legal possession. For example, L was in adverse possession of house, leased it to T, but mentioned to T that if the true owner showed up, T would have to move. For this reason, L charged T rent that was 30% below market rate. T is deemed to have implicitly waived his right to legal possession. Ts remedy if L fails to deliver legal possession: can terminate the lease, or waive his right to receive legal possession and continue occupying property and paying rent. Ls failure to deliver legal possession is not excuse for T to withhold rent if T is in actual possession (example, Problem 2, page 388, T, who had actual possession of leased property, could not withhold rent for reason that L had entered lease of the same property with another tenant).

3.

4. 5.

B. Landlords obligation to deliver actual possession 1. Traditional common law (and minority US jurisdictions: American Rule): landlord has no obligation to deliver actual possession (example, Hannan v. Dusch, page 384, L had no obligation to evict holdover tenant from prior lease). a. Ts remedies T has the same right against the wrongful possessor as L. He can treat the holdover as a trespasser and evict and recover damages, or he can renew the holdover for a new term, receiving the rent from the holdover. These may not be satisfactory remedies to T.

2. 3.

Majority US jurisdictions: landlord has obligation to deliver actual possession. Rationale: L is in a better position to know the possibility of holdover. And tenant cannot evict until HIS lease is supposed to start. Hypo: L leases from T hunting lands but cannot get to the land because neighbors will not allow it. Has L satisfied the duty. Answer: L has failed to deliver actual possession under the English rule but this is satisfied under the American rule

16

4.

Hypo: if trespasser enters on the 2nd day of the lease then the lessee has the responsibility to remove trespasser.

C. Tenants Duty to occupy? 1. Is he required to actually occupy? a. Generally no. if he pays rent and the lease is silent on occupying, then he doesnt have to. b. EXCEPTION: i. Explicit provision in the lease ii. The landowner gets a percentage of the rent. The courts are willing to find that the parties have implicitly agreed that the tenant has a duty to occupy. (1) If the landlord charges you rent close to market value, but be also takes 3% of your business, then courts arent willing to find an implicit agreement. iii. Waste (1) Ts failure to occupy the leased property would result in a waste of the property value, or would decline the property value. D. Landlords duty to keep the rental property habitable 1. The doctrine of constructive eviction a. In every lease, there is an implied duty on the part of the landlord to provide quiet enjoyment of the leased property. b. If this is violated then constructive eviction has happened c. What constitutes constructive eviction? i. The interference must be such as to render the premises substantially unsuitable for the purpose for which they are leased (1) Substantially unsuitable is a factual issue determined on case-by-case basis (2) A reasonable person standard applies here. (a) Failure to provide heat in the month of November for a property located in Boston is more likely to be a constructive eviction than failure to provide heat in November in Florida. ii. (E.G., Reste Realty Corp. v. Cooper, page 422, basement office flooded when it rained). iii. Other examples (1) Failure to supply heat (2) Strong and offensive odor (3) Leasing other parts of same bldg to other Ts engaged in illegal/immoral activities rendering Ts premises unsuitable for a decent family d. What if the lease has a provision that says, T has inspected the premises and accepted the premises in their present condition? i. T is barred from claiming constructive eviction [caused] by L, IF: (1) The defects were not latent AND (2) The defects that caused the problem were part of the leased premises (3) Landlord has made a promise that he would remedy the defects by the time that the lease begins. ii. (E.G., in Reste Realty Corp. v. Cooper, page 422, the driveway caused the flooding. Since the driveway was not part of the lease, the above provision had no effect). iii. Reasoning (1) Driveway, its surfacing, the exterior wall, or foundation cannot be considered included as part of the premises under the lease (2) Even if included w/i premises, nothing in evidence suggests that condition in driveway/wall would be noticeable upon inspection

17

(3) The condition was probably a latent problem, which the prior-L (if aware) had a duty to disclose, regardless of lease provisions (4) Even though was aware of the problem prior to signing 2nd lease, had recd assurances that problem would be remedied. In fact, prior-Ls on-site mgr. had tried to remedy by resurfacing the driveway but the attempt proved unsuccessful (5) Interference not reqd. to be permanent; it is only reqd. to be substantial iv. RESULT/HOLDING (1) L did breach covenant, thereby justifying Ts vacating the premises (2) T did not waive right to vacate b/c T vacated w/i a reasonable time of the constructive eviction coming into existence (3) T did not assume responsibility for defect b/c at first T was unaware of latent defect and, when aware, prior-L promised to remedy (4) Premises rendered Substantially unsuitable for the intended use. Permanent rain damage is not necessary. As long as it happens regularly. e. Tenants remedies when constructive eviction established: i. FIRST, Tenant MUST vacate premises; and (1) Minority of jurisdictions do not req. such vacating as pre-condition (a) T may have a hard time finding a suitable, affordable, alt. place to live (b) T may be unsure whether Ls conduct constitutes constructive conviction (c) Vacating pre-condition is inconsistent w. implied warranty of habitability doctrine, which does not req. T to move out ii. T, by terminating lease, is relieved from further obligation to pay rent; and iii. Tenant may recover damages to his belongings and business (1) Damages equal difference b/w rents paid and the FMV of premises in its defective state (2) Consequential damages are available for things such as hotel expenses while looking for another place to stay iv. T may seek injunction to compel L to fix the defect v. Note: If T does not respond to Ls constructive eviction by abandoning the property within a reasonable time, he is deemed to have waived his right of abandonment. 2. The doctrine of implied warranty of habitability a. Applies to residential leases.-a lot more powerful in providing protection to the tenant. i. In each lease, the landlord has made an implicit warranty that the property is safe, clean, and fit for habitation.

b. What constitutes breach of warranty? The claimed defect has an adverse impact
on the safety or health of the tenant, making the leases premises uninhabitable in the eyes of a reasonable person (example, Hilder v. St. Peter, page 431, bathroom toilet was clogged, window was broker, the door had no lock, leak of water pipes, etc). Breach may exist even though housing code is not violated (this is because there are a lot of problems that are not addressed in the housing code). i. The tenant is entitled to remedy at this point. (1) Tenant is not required to move out of the property in order to claim under this implied warranty. (a) This doctrine is the applied in most jurisdictions to residential leases only. (2) Need to give a reasonable amount of notice in order for the landlord to fix the defect.

18

ii.

iii.

iv.

v.

(a) Failure to give adequate notice to fix the defect waives the tenants right to get remedy under this doctrine. Note: In order to bring a cause of action for beach of the implied warranty, the tenant must first show that he or she notified the landlord of the deficiency or defect not known to the landlord and allowed a reasonable time for its correction. This is a reasonable person standard (1) If a reasonable person in the community where the property is located would reasonably believe that the property could adversely affect the safety or health of the new tenant, then this fails. The subjective belief of the new tenant is irrelevant. Ex. (1) On-site laundry- no not a breach (2) Air-Conditioning- yes a breach if in summer high heat (3) Sex offender- not a breach of implied warrant of habitability. They will look at the likelihood of a recurring offense. (4) Violation of housing code- this should be one factor and it is a strong factor but it is not conclusive evidence in itself *big difference between implied warranty and constructive eviction is that plaintiffs do not have to vacate premises.

c. This is strict liability on the part of the landlord landlord is liable even though
causes of defects are beyond his control. i. Note 4a page 439

d. The implied warranty of habitability cannot be waived by the tenant. A landlord


cannot nullify the implied warranty of habitability on a dwelling by giving his tenant a discount in rent. This is because courts recognize the limited bargaining power of tenants. e. Tenants remedies upon landlords breach of warranty i. Refund of the difference between true market value of the property and the value of the dwelling as warranted (which usually is determined by reference to the rent). (1) Percentage rental deduction. If 20% of the property is not usable because of the defect, then theyll deduct 20% of the market value after taking into consideration the market value with defects. (a) X is worth $100. The market value of X with defects is $50. The landlord will then take 20% off $50. ii. Future rent is excused until defects have been corrected. (1) What if only one bathroom is defected, but everything else in the house is fine? (a) To avoid the dilemma, a tenant is allowed to withhold the entire amount of rent. (b) If later on the court finds that you are at most required to withhold $200, then you must pay the rest of the rent to the landlord. iii. Deduction from any future rental obligations reasonable expenses incurred by tenant in correcting defects. iv. Punitive damages may be awarded for willful or wanton or fraudulent breach by the landlord v. Special consequential damages (example, tenant had to incur the cost of staying in a hotel while looking for another apartment).

19

f. Does the constructive eviction model becomes moot if you use the implied warranty? i. No. not every state has recognized the implied warranty. ii. The doctrine of implied warranty only applies to residential leases while constructive eviction applies to both residential and commercial leases. 3. Landlords tort liability for failure to keep the rental property safe a. Common law (and most US jurisdictions): landlord is not liable in tort. Exceptions:

i.

Public use exception, which imposes tort liability on landlord if the following conditions are met: (1) L leases to T for a use open to the public; (2) The defect in question exists at the outset of the lease; and (3) L knew or should have known of the defects; and (4) L knew or should have known that T could not reasonably be expected to remedy the defect or guard against injury, and (5) The injured person is a member of the public rather than T. If T were the injured person, he would have to fall into some other exceptions. Latent defect exception, which says that landlord is liable for latent defects if at the outset of the lease, L knew or should have known the existence of the defect, and T neither knew nor had reason to know the existence of the defects. care in keeping common areas safe. (1) Is landlord responsible in torts for criminal conduct of 3rd parties? Courts split. Some courts say landlords duty extends only to defects on the property, while others impose a duty of reasonable care on landlord to provide security where landlord has been put on notice of repeated criminal activities in the area.

ii.

iii. Common area exception, which says that landlord has a duty to use reasonable

b. Effects of tenants waiver: Lessor shall not be liable to tenant or to any other
person or to any property for damage or injury occurring on or owing to the condition of the leased premises, or any part thereof, or in the common areas thereof, and tenant agrees to hold lessor harmless from any claims for damages no matter how caused. i. Validity upheld in commercial leases. ii. Court split in residential leases. VI. Tenants obligations and landlords remedies A. Tenants duty to repair 1. Common law: tenants have duty to repair except for extraordinary and substantial repairs. 2. Common law duty may be altered in lease agreement. A general repair clause obligates tenant to repair in order to preserve the property in the same condition as it was at the outset of the lease. An unqualified general repair provision imposes duty on tenant to rebuild the leased premises after complete destruction. The doctrine of implied warranty shifts repair burden to landlord.

3.

B. Tenants duty to avoid affirmative activities that substantially damage the property

20

1.

Substantial damage must be extraordinary in scope and effect. A factual issue that may give rise to disagreements.

C. Landlords options if tenant holds over at the end of the lease 1. Common law gives landlord two options eviction (plus damages) or consent (express or implied) to the to the extension of the lease for another term. Nature of new lease: if L elects to have the lease extended for another term, most states treat the new tenancy as a periodic tenancy. a. When the landlord talks with the tenant he has to be careful. He may be deemed to have made a choice when he really didnt.

2.

3.

Holdover must be out of control by Tenant: Example: L leased the property to T for 3 years for $1,000 per month. T gave good notice to L of his moving out at the end of the lease. On the last day of the lease, T suffered a heart attack and was hospitalized for a week. Question: can L elect to have the lease extended for another term of 3 years? Answer: No, the situation was beyond control of T, and thus T was not a holdover. Note: T will be a holdover if after the condition has terminated, he still does not move out. What has landlord elected? Example: Crechale & Polles, Inc. v. Smith (page 369, L denied existence of oral agreement to extend the lease and demanded T to vacate the property. However, about 2.5 months after T held over, Ls lawyer wrote to T to notify him that L was treating the holdover as a renewal of the lease for another term). L has elected to treat T as holdover tenant and thus, if L accepts check instead of evicting T in time, he is deemed to have agreed to an extension of the lease on a month-by-month basis. Example: L leased an apt to T for a term of years. After the K has expired, T held over for another 2 years. During that time, T tendered rent on a monthly basis, but each time L refused to accept the rent and demanded T to move out. However, L did nothing else to evict T. Question: has L elected to have this lease renewed for another term? a. Answer: the courts split. Some have held that consistently refusing rent equates to not extending the lease. Others have held that by failing to take adequate action to evict the T, the L has elected to extend the lease.

4.

5.

6.

When must L make the choice of eviction or extending lease? Reasonable amount of time. a. Reasonable amount of time factual issue that varies b/w situations and Ks i. Reasonableness varies from case to case.

D. Landlords self-help measures when T holds over or fails to pay rent or perform other lease obligations 1. Remedies Available when T defaults (on rent or other lease obligations) a. terminate and repossess the premises i. equitable relief from forfeiture of lease if rents are paid w/i a reasonable period b. Also, L can sue for back rent and other damages

2.

Common law approach: Landlord may use self-help measures if two conditions are met: a. Landlord is entitled to repossession, AND i. E.G. T holds over after the lease term or T breaches a major term of the lease b. Method of reentry is peaceable

21

i.
ii.

iii.

(E.G., Berg v. Wiley, page 403, L changed lock while T was away, court found method not peaceable). Peaceable (1) Voluntary transfer of possession (a) Landlord appears on the property and tells the tenant that he is retaking possession of property because he has breached and tenant leaves on his own. (2) L cannot use force or threats, AND L cannot use a ruse or stratagem to trick T out of possession Examples of non-peaceable (1) Picking a lock (2) Entry through an open window (3) By removing the doors (4) By changing locks

3.

Modern view: L cannot use self-help measures, must resort to judicial proceedings (example, Berg v. Wiley, page 403, the court found Ls entry not peaceable but rested opinion on prohibiting self-help measures entirely). Courts are more willing to apply this approach in cases of residential leases. Many jurisdictions have established summary proceedings that provide judicial remedy for L in a few days a. Such proceeding in Ohio is called Forcible Entry and Detainder b. What if the lease provides landlord may repossess the property upon Ts default? i. ii. iii. iv. v. Courts split. Some regard this an indication T has waived his right Majority and modern courts hold the view that the policy against self-help measures cannot be contracted away **However, courts are likely to limit Ts ability to waive to only commercial leases There is a disparity in the bargaining power.

4.

Residential v. Commercial Lease a. Some STs allow self-help in context of commercial leases, but not residential leases. Why? i. The psychological impact due to loss of residence is bigger than that due to loss of commercial space ii. The bargaining powers of the parties in a commercial lease tend to be equal Example: Berg v. Wiley (MN 1978) [p. 403] a. Facts i. The lease (1) L and Ts predecessor entered into lease in 1970, for 5 years (2) T should not alter the premises w/o first seeking consent from L. (3) L may repossess the property upon Ts default/non-compliance w. leases terms" ii. T remodeled, w/o consent, and L did not like the remodeling iii. L gave notice that T must undo the remodeling, or else L would evict T in 2 weeks iv. At deadline, T dismissed employees, closed the restaurant, and placed sign closed for remodeling v. L attempted to change locks, but T returned in time to intervene and prevent vi. L, hanging around property, become concerned about destruction of property and called police

5.

22

vii. Police mediated an agreement b/w parties to maintain status quo, until each party consulted counsel b. Issues i. Whether T abandoned/surrendered the premises ii. Whether Ls reentry was forcible and wrongful as a matter of law (despite lease provision permitting Ls self-help) c. Held i. Sufficient evidence to support jury finding that T did NOT abandon/surrender the premises ii. T.Ct. determination that, as a matter of law, Ls reentry was unlawful was correct 6. Ls liability for conversion/destruction of Ts personal items a. E.G. L sued to evict 2 tenants for failure to pay rent. Before the eviction proceeding and when the Ts were out-of-town, L removed the Ts personal property from the premised and placed the property on the street w. a sign saying Free Take. Ts were, by judicial proceeding, evicted. But, the Ts sued to recover damages for the value of the personal items. [See Chryar v. Wolf, 21 P.3d 428 (CO Ct. App. 2000)] i. Ts could recover for the value of the personal property b/c the L had wrongfully taken the property. (1) If the L had simply placed the personal property into a storage room, then no damages b/c Ts could simply collect property

E. Landlords choices when tenant abandons lease 1. Accept abandonment and terminate the lease, releasing T from future lease obligations/rent (T must pay past due rent and for any damages to prop). a. What constitutes abandonment by tenant? May be subject to dispute (example, Berg v. Wiley, page 403, closed for remodeling sign and discharge of employees not indication for abandonment) b. After tenants abandonment, L may rent out the property again, and tenant is not entitled to any higher rent that the landlord is able to charge. c. Why would you want to just accept the property? i. If you reject the abandonment, then the tenant is kept on the hook. The landlord can go find someone else, and if the new tenant defaults, then the landlord can go after both of them. To accept the abandonment doesnt seem so appealing now.

2.

Landlord may refuse to accept tenants surrender of the premises and sue tenant for damages. a. Lease is not terminated. L may sue for damages based on the Ts anticipatory breach. b. If the landlord rejects the abandonment, can the tenant come back later? i. If the landlord has already leased the property, then no, even though he can still be liable for rent if the new tenant defaults. ii. If there isnt a new tenant then the courts are split. (1) Some courts say that the tenant should still be able to come back because technically the property still belongs to him. c. Is landlord required to mitigate damage? i. Traditional common law: no duty to mitigate. Why? (1) Lease is mere a conveyance of real property than a K. Thus, L should not be concerned (or obligated) by the consequences of T abandoning his property interest. T cannot by his own wrongdoing impose a duty on L. (2) Law does not want to encourage abandonment of lease by T b/c it leads to waste and vandalism.

23

ii.

iii.

(3) Ls mitigating effort may be deemed as acceptance of Ts abandonment and thus L would be forfeiting recovery from T for future rents should L fail to rent out the premises (4) Requiring L to mitigate damages causes complexities in litigating re whether L has acted reasonably to mitigate damages (5) Unfair b/c L loses chance to rent out another vacant apt in a multi-unit bldg Modern view (majority US jurisdictions): L has duty to mitigate by renting out the premises in residential leases. (1) This is due to the increasing view that a lease is a contract and as such imposes duty to mitigate on the non-breaching party (2) (E.G., Summer v. Kridel, page 410) (a) T signed 2-yr lease, but notified L prior to start of lease term that she would be unable to fulfill lease (b) L failed to re-let until more than a year after Ts surrender (c) L sued T to collect back rent (d) HELD: L failed to take reasonable action to mitigate damages (e) As soon as landlord accepts surrender of tenant then the contract ends [A] Therefore landlord should not accept tenants surrender (3) Burden of proof: (a) Most jurisdictions require L to show he has used reasonable effort to re-let the prop (b) Minority req. T b/c in K law breaching party has burden to show lack of reasonable effort to mitigate by non-breaching party (4) Why require mitigation? (a) Lease is a K rather than a mere transfer of possessory interest in real property. Thus, K law req. mitigation should apply to leases (b) T has a significant liberty interest in being able to move out before end of lease term, so long as Ls financial interests are fully protected (c) Each apt is unique and suits different tastes of different customers. Even if L is not reqd. to re-let the premises, he still may not be able to rent out another unit (d) **Uniform Residential L & T Act (URLTA), adopted in about the STs does impose such duty on L in residential tenancies (e) Policy against waste [A] Promote a more productive use of the property. (5) Does mitigation duty apply to commercial leases? (a) Courts split (b) Jurisdictions that extend the duty to commercial leases are governed by the ordinary K principles (6) If new T defaults during term of 1st Ts lease, then 1st T is liable for the default (a) But L cannot double-collect IF L has duty to mitigate: (1) If L has failed to meet the duty: (a) his damages (recoverable in suit) are reduced by the FMV of rental of the property (b) Note: this includes the situation where L has failed to put up reasonable effort to re-rent the space at its FMV [A] E.G. L hastily re-rented the property for $100, while the FMV was $200 and the original rent was $150. In this situation, L would not be entitled to recover any damages b/c by reasonable effort L could have rented the premises for at least $150

24

[B] EX 2: there were 8 months left in the lease. Rent was $100 a month. L didnt attempt to mitigate, but there was evidence to show that he could have gotten $90 a month. (ANSWER: $80) (2) If L has satisfied the duty: (a) L damages are the difference b/w the agreed rent and the rent that L could charge form the new-T (b) If he tried reasonably to find new-T but failed, then Ls damage is the agreed rent (c) Note: Law does not compel L to re-let the leased premises. Indeed, L may sue before he starts any effort to find a new-T. However, in such situation, FMV of rental must be deducted from the damages compensation d. Waiver by T of Ls duty to mitigate i. Enforceable ii. Rent acceleration clause intended as a waiver of the duty to mitigate (1) Enforceable, unless the payment upon acceleration is unreasonably high relative to the estimated damages (2) Note: scholars have pointed out that in assessing reasonableness a court could essentially impose the duty to mitigate on the L again, thereby rendering the acceleration clause meaningless e. How can L avoid having his mitigation effort be construed as acceptance of Ts surrender? i. L can simply communicate to T, promptly and effectively, the message that L is not accepting Ts abandonment and is merely re-letting the premises on Ts behalf f. Landlords remedy if he refuses surrender by tenant: may recover the difference between the rent specified in the lease and the amount he collects from new tenant based on market value. g. Landlord re-letting of premises may lead to belief that he has accepted tenants abandonment. To avoid this, landlord should make it clear to tenant that I am reletting the premises on your account. You are still liable for the difference. F. Tenants duty to repair 1. Common law: tenants have duty to repair except for extraordinary and substantial repairs. 2. Common law duty may be altered in lease agreement. A general repair clause obligates tenant to repair in order to preserve the property in the same condition as it was at the outset of the lease. An unqualified general repair provision imposes duty on tenant to rebuild the leased premises after complete destruction. The doctrine of implied warranty shifts repair burden to landlord.

3.

Estates and the Rule against Perpetuities


I. Scope of Coverage A. Present possessory estates;

25

II.

1. Person who holds the interest can possess it right now. a. Fee simple absoute b. Fee Tail c. Life estate d. Defeasible fees B. Future interests; 1. No possessory interest right now, but you may have one in the future. C. The Rule against Perpetuities D. Note: Practice is key to mastering the concepts summarized in this outline Present possessory estates A. Estates v. Interests 1. 4 Types of Estates (& their durations) a. Fee Simple (forever) b. Fee Tail (until original grantees lineage dies out) c. Life Estate (for the life of grantee) d. Term of Years (fixed period measure in years, months, or days; or a date certain) e. ** 1-3 are Freehold Estates; 4 is a Non-freehold Estate (although the distinction is not very relevant today) 2. Interests a. All estates are interests, but not all interests are estates B. Fee simple absolute (to A and his heirs, to A, etc.) 1. Definition a. Absolute ownership (title) in real property for perpetual duration without limitations i. Absolute ownership no conditions attached (1) Most unrestricted and longest estate. b. (A.K.A. complete ownership until the end of time) 2. Creation of fee simple absolute interest a. Common Law: to A and his heirs b. Modern View: to A and his heirs is not reqd. i. Other acceptable phrases include to A, to A forever, etc. ii. But still most lawyers stick to the tradition of using and his heirs in order to grant fee simple absolute. 3. Transferability of fee simple absolute interest a. Inheritable under intestacy statutes. Thus if the owner of a fee simple dies, the property passes to the people deemed to be his heirs under the intestacy statute of the state where the land is located. b. Ownership is freely transferable by alienation, will and inheritance i. A salient feature of fee simple estate ii. ANY restriction on transferability is void c. Devisable by Will i. E.G. O conveys land to A and his heirs. A dies, but in his will, A leaves the land to a friend B. (1) Does As son C have any claim in the property on the ground that he is the only heir to A? (2) NO. A is free to transfer the land, by will, to any individual, and the phrase and his heirs does not create any ownership right in C. C. Fee tail (to A and the heirs of his body) 1. Example: a. O grants to A and the heirs of her body, and if A dies without heir, then to B and his heirs. i. Creates a series of life estates in A, and upon As death, to As children, and so on. (Traditionally it was to As eldest son. The law has relaxed in allowing daughters to share this interest)

26

(1) Direct blood line. A has fee tail not fee simple. If at the time A dies, A has a son A1, A1 gets the fee tail. B gets nothing. B gets the property only after As line of blood has run out. v. Note: Bs interest is called remainder. 2. Elimination of fee tail in US jurisdictions: a. In the US today, fee tail has been largely abolished except for in a few states (ME, MA. RI, and DE) b. Even in states that still keep fee tail, the fee tail can be destroyed by an ordinary conveyance of fee simple absolute by the fee tail holder during his life time (not by will or intestacy). i. Example: O grants house to A and the heirs of her body, and if A dies without heir, then to B and his heirs. A conveyed the property by deed to C and his heirs. C has fee simple absolute interest. (1) When A dies without heirs, B does not have any interest in the property. (2) C has fee simple. c. What interest results in States that have abolished fee tail? i. Two main approaches: (1) By statute, fee tail is converted into fee simple or the largest estate the grantor owns (a) (E.G.: O has life estate in land, and conveys to To A and the heirs of her body. A holds a life estate b/c that is biggest estate that O owned) (2) By statute, fee tail is converted to fee simple but subject to executory limitation if the fee tail has a remainder in someone else (a) (E.G.: To A and heirs of his body, then to B and his heirs converts to: To A and heirs, but if A dies w/o issue, then to B and his heirs). So if A has an issue, then the As will have fee simple absolute forever. D. Life estate (to A for life) 1. Reversionary v. Remainder Interest a. If O grants to A for life, then O maintains a reversionary interest, which takes effect upon As death b. If O grants to A for life, remainder to B and his heirs, then B holds a remainder interest, which in this case would be a fee simple estate i. Unless it says remainder to B for life. That would be another life estate in B. 2. Life estate in a group of people: a. O grants land to my children for their lives and the remainder goes to Church. O has 3 children. Child 1 dies. i. Does the church get 1/3 of interest now? ii. Answer: most jurisdictions hold that child 1s interest goes to the remaining children. Church will get land when all children die. 3. Ambiguous grant: a. Courts prefer to find fee simple over life estate w/o contrary evidence i. In cases of ambiguity, courts will choose fee simple because it is less restrictive in transferability and practice than the transferability in a life estate. ii. Courts are also willing to infer that the grantor has given 100% of their will. b. Courts presume that grantor has granted his entire interest rather than partial interest w/o contrary evidence c. Restrictions inconsistent with fee simple are voided i. E.G.: White v. Brown, p. 190 (1) I wish White to have my home to live in and not to be sold. I also leave my personal prop to Sandra White Perry. My house is not to be sold. ii. iii. iv.

27

(2) Decedents other legal heirs claimed that this will created a life estate, not a fee simple absolute. Lower courts agreed. (3) TN Supreme Ct. held that the will created a fee simple by finding: (a) That applicable TN statute provided that a fee simple absolute was passed unless the testator clearly intended to devise a life estate. (b) Fee simple absolute was presumed (even w/o words of inheritance), if there was no language that could be construed to create a remainder. (c) Thus, the restraint on the sale of the house was insufficient to overcome the presumption that a fee simple had been devised, and testamentary restraint on alienation was void (d) The court focused that White was the only person that the deceased was associated with up to her death [A] She was the only one who actually took care of her prior to her death. There was no evidence that the nieces and nephews were in the picture. She didnt even mention the nieces and nephews in her will at all. (e) Dissent focused on the restrictions 4. Transferability of life estate: a. Freely transferable, but transferred interest is limited by the life span of transferor/life estate holder b. E.G. A holds life estate in the house. A writes to B, I want you to know that I want you to have my house for as long as you live. Then, A dies. i. Question: Does B have a life estate in the house? ii. Answer: No, Bs interest in the house terminates when A dies c. NOTE: Life Estate holder can transfer fee simple, only if he obtains the consent of all the remainder/reversionary holders 5. How an a life estate holder convey a fee simple? a. By obtaining consent from all holders of the remainder interest. 6. Undesirable problems associated with life estate: a. Hard to sell or lease property for an extended period of time b/c buyers/lessees can only purchase/lease property for length of grantees life b. Judicial response to the inflexibility of a life estate: i. Courts sometimes (though not often) order the sale (or a partial sale of prop) and divide the proceeds b/w the life estate holder and the remainder ii. Key factors courts consider: (1) Whether the sale is in the best interest of all parties (including whether sale is necessary to avoid waste), AND (2) Whether sale is the only practical way to provide material comfort for the life tenant and to preserve asset value for the remainder interest iii. E.G. Baker v. Weedon, page 197. (life estate v. remainder) (1) Widow, w. life estate in farmland, tried to sell land, which was expected to appreciate in value soon, to help subsidize her living expense (2) HISTORY: T.Ct. allowed her to sell land and thus divested remaindermen of their contingent interest. (3) ISSUE: should the property be sold to sustain a comfortable living of the life estate holder? (4) RESULT: MS S.Ct. revd and remanded (a) You still have to take into account the interest of the grandchildren, despite the presumed intent of the deceased or the equitable relief of Anna. (5) HELD:

28

(a) Deterioration & waste of property is not exclusive & ultimate test in determining whether a sale of land affected by future interest is proper, (b) Ct. should also consider whether a sale is necessary for the best interest of all the parties, i.e., the life tenant & the contingent remaindermen (c) Intent of the husband: [A] He seemed to speak very highly of her because he wanted her to be taken care of, and the house could then go to her children, even if they werent his. (6) RATIO: (a) Parties should convene to try and resolve the issue independent of court system (b) Perhaps, a partial sale of the property would be a better resolution (c) S.Ct. remanded the case to allow for a resolution that protected the interests of all affected parties c. Modern life estate equitable life estate (as opposed to legal life estate): The mechanism works like this: (how to resolve the tension between a life estate and the remainder interests) i. O, who intends to grant a life estate in A, puts property in trust, with beneficial interest held by A for life, and remainder to O/3rd P ii. The trustee holds legal title to the property in the form of fee simple absolute, while A (beneficiary) holds the beneficial interest. iii. Trustee can sell land and transfer fee simple absolute interest to buyer if trustee believes that such sale would be for the benefit of all equitable owners (1) Note: Trustee owes equitable owners fiduciary duty and thus is required to act for their best interest. iv. For added flexibility, A can be both the trustee and the beneficiary. (1) To A as trustee to hold land for the benefit of A for life, and then to B outright and free of trust v. Language used (1) To T, as trustee, to hold land for the benefit of A for life, and then to B outright and free of trust vi. Presently, every life estate is granted in the form of a trust. 7. Duty of legal life estate holder to avoid waste a. Generally, life estate holder is reqd. to use the property in such a way as to avoid waste b. Definition of waste: i. Action of the life estate holder that permanently impairs the propertys value or the interest of the remainder holders. c. Different types of wastes i. Affirmative (or voluntary) waste (1) Life estate holder acts affirmatively to damage the property ii. Permissive (involuntary) waste: (1) Life estate holder fails to act reasonably to prevent the deterioration of the property (a) E.G. failure to treat termites iii. Ameliorative waste: (1) Life estate holder changes the principal use of the property, and thereby increases the value of the land or at least does not decrease the value. (2) Cause of action arises only if the change is inconsistent with the intention of the grantor, and the property may still reasonably be used in the fashion that the grantor had intended

29

(3) E.G.: To A for life, to be used as a farmland, remainder to B and his heirs. (a) If A builds an office building on the land and as a result the value of the land increases, then B, who loves farming and has been dreaming of owning his own farm, may still sue for ameliorative waste b/c the office building is inconsistent with the intention of the grantor and the land may still be reasonably used as a farmland even though using it as a farm land gives only a fraction of the value of using it as an office space. (b) The law wants to promote more productive use of the land. [A] In absence of any language to the contrary, the law does not presume that the grantor only wants the land to be used the way it was used when it was granted. (4) If things/conditions beyond the grantees control cause the property to be unusable for the purpose selected by the grantor, then some courts may allow the grantee to convert the property to another use (but the courts are reluctant to do so) d. Duty to take insurance on the property: Jurisdictions split. i. In states that do not require insurance, the insurance proceeds belong to the life estate holder who paid the premium. ii. In STs that req. insurance, the proceeds are divided b/w life estate holder and the remainder man in proportion to their respective value of the property.

E. Defeasible fees (fee simple interest that is interrupted by the occurrence of some events)
NEED TO MEMORIZE THE EXACT TERMINOLOGY OF THESE DEFEASIBLE FEES. 1. Fee simple determinable a. Conveyance of a fee simple interest for a limited duration. b. Grantees property interest automatically terminates upon occurrence of specified event/nonevent i. (e.g., to A so long as A refrains from smoking) c. Words commonly used to create fee simple determinable: i. So long as, ii. While, iii. During, iv. Until, v. Unless, (but can also be interpreted as a condition attached) vi. And any other word that limits the duration of the grant d. Freely transferable, but the nature of the estate stays the same i. But the transferor will be bound by the same limits. ii. If A transfers to B, but if the next day, A picks up smoking, B loses every penny that he has paid. He loses the interest completely. The property goes back to O e. Possibility of reverter chance that the property might return to the grantor if condition subsequent happens f. Relationship i. A fee simple determinable is a present possessory estate followed by a possible reverter in the grantor (1) Sometimes the possibility of reverter is expressed in the deed/will creating the fee simple determinable (2) If not expressed in will/deed, then it will be implied as part of the nature of a fee simple determinable ii. Grantee: holds property in a fee simple determinable

30

Grantor: if remainder is in the grantor, then his remainder interest is called possibility of reverter g. NOTE: i. Grant does not have to specifically mention grantors residual interest. (1) Reversion back to grantor occurs automatically by operation of law upon the occurrence of the specified event/nonevent. ii. Possibility of reverter is a future interest 2. Fee simple subject to condition subsequent a. Definition: i. Grantor intends to convey a fee simple absolute but has attached a condition to the grant (1) If a specified future event/nonevent happens, the grantor MAY revoke the grant and regain his fee simple absolute estate (2) E.G. to A, but if A picks up smoking again, O has the right to terminate As interest b. Condition Subsequent an event whose occurrence/nonoccurrence will terminate the estate c. Words commonly used: i. Provided that, ii. But if, iii. On condition that, iv. Provided, however d. Difference from fee simple determinable i. A complete grant (of unlimited duration) but with (possibly forfeiting) strings attached ii. The grantor must exercise his right of entry in order to terminate grantees interest. (1) What constitutes exercise right of entry? (a) Words alone are not enough. (b) There must be additional actions. (2) Until grantor exercises his right of entry (power of termination), holder of fee simple subject to condition subsequent continues to own prop (3) Also see mahrenholz on 47 of supp. e. Freely transferable, but the nature of the estate stays the same. f. If A dies without ever smoking, then the condition is never satisfied. i. It will go to his estate instead of back to O. g. In case of ambiguity in the grant, courts try to construct grantors intent by looking for evidence that indicates whether upon the occurrence of specified event, the termination of granted interest occurs automatically or requires action by the grantor. i. Fee simple subject to condition subsequent is preferred over fee simple determinable. h. What conduct by grantor constitute exercise his right of entry? i. Generally, substantial steps to recover possession and title of the property (1) E.G. (sufficient assertion) posting signs on the property, starting to pay tax on the property, etc. (2) E.G. (mere communication of intent to exercise right) (a) To A, but if A starts smoking, the property goes back to O. When A picked up smoking, O sent him a letter saying that I would like to exercise my right to terminate your interest in the land. But, O did nothing more. Is this sufficient assertion of right of entry? [A] Answer: No, Os actions were insufficient i. Ambiguous Language

iii.

31

j.

In constructing grantors intent, look for whether interest is terminated automatically upon occurrence of specified event or requires action on the part of the grantor. ii. E.G. to A so long as he refrains from smoking, but if A smokes, grantor has the right to enter and retake possession. (1) The ambiguity (a) Used so long as usually indicative of limited grant and thus fee simple determinable, AND (b) The phrase has the right indicates the interest does not terminate automatically upon As smoking, but rather O must exercise his right (2) Fee simple determinable or fee simple subject to condition subsequent? (a) Answer fee simple subject to condition subsequent (b) NOTE courts prefer to construe ambiguous language as fee simple subject to condition subsequent iii. Why does it matter if we pick fee simple determinable v. fee simple subject to condition subsequent? (1) Importance lies in whether or not interest is terminated automatically or requires further action by grantor (a) If evidence shows that the interest will come to end automatically, courts are more willing to treat the interest as a fee simple determinable. (b) BUT in cases of absolute ambiguity, courts are more likely to rule in favor of a fee simple subject to condition subsequent. (2) E.G. adverse possession cases: in 1990, O grants to A so long as he refrains from smoking, but if A smokes, grantor has the right to enter and retake possession. In 1995, A starts smoking. O has done nothing to retake possession of the property. The state has a SOL of 10 yrs. In 2006, O wants to retake possession of the property. A claims adverse possession. What result? (a) Answer: [A] if the court were to find that the interest conveyed to A was a fee simple determinable, then SOL would start to run as soon as A started smoking [B] If the court were to find that the interest conveyed to A was a fee simple subject to condition subsequent, then SOL would not start to run b/c As interest in the property does not terminate automatically. Instead, it only terminates when (and if) O sufficiently asserts his right of entry. [C] Courts look at the intent to see if it should terminate automatically. [D] Reminder, if ambiguous, courts prefer fee simple to condition subsequent (3) To A and his heirs, provided that the estate shall cease if liquor is sold, used, or stored on the property. (a) provided that means fee simple subject to condition (b) shall cease indicates that once this condition happens the interest will end automatically. Transferability of fee simple subject to condition subsequent: i. Freely transferable, but the transferred interest is still the same fee simple subject to condition subsequent

i.

3. Fee simple subject to executory limitation a. Definition

32

O conveys a fee simple and upon the occurrence of specified event, the fee simple interest is forfeited in favor of a 3rd P (not the grantor) ii. E.G. O grants land to A so long as he refrains from smoking, otherwise to B and his heirs. iii. E.G. O grants land to A, but if A shall ever smoke again, to B and his heirs. b. Interests associated with fee simple subject to executory limitation i. The grantee: fee simple subject to executory limitation ii. 3rd P who holds the remainder: executory interest c. Automatic forfeiture upon occurrence of specified event i. Prevailing view: automatic forfeiture in favor of the 3rd P, regardless of whether the divesting condition is phrased in the form of a determinable fee or a fee simple subject to condition subsequent

i.

d. Property goes to a 3rd person rather than the grantor upon occurrence of specified

event (example: O grants land to A so long as he refrains from smoking, otherwise to B and his heirs. As interest is called fee simple subject to executory limitation, Bs interest is called executory interest). third party remainder upon the occurrence of the divesting event.

e. Fee simple subject to executory limitation is automatically divested in favor of the


4. Distinguish conditions in defeasible fees from covenants a. Different Consequence i. If the granting language contains a condition or limitation: its occurrence leads to forfeiture of interest. (1) If it happens it interrupts the presently possessor of the interest. ii. If granting language contains a covenant or promise and the covenant or promise is breached, then the grantor may sue for an injunction or damages, but the grantees interest is not subject to the risk of forfeiture iii. E.G. (note 4, p. 215): O conveys land to A, who promises on behalf of himself, his heirs, and assigns that the land will be used only for residential purposes. If A erects a slaughter house on the land, O can sue A for an injunction or for damages, but As interest is not forfeited iv. Point out the ambiguity between defeasible fee and covenant and then argue both (1) Is it a promise? Then does not revert back to O (2) OR is it a restriction? Does revert back to O (3) Courts will most likely focus on the word promise and not award property back to O (a) This means that A holds promise in fee simple absolute but A can still sue for an injunction or damages. 5. Fee simple determinable v. Fee simple subject to condition subsequent a. Fee simple determinable is of limited duration, not perpetual duration b. Fee simple subject to condition subsequent, potentially, is of unlimited duration (provided that grantee fulfills reqd. condition) 6. The validity of the restrictions in the grant a. Restrictions in defeasible fee b. Restrictions in life estate c. The validity of restrictions in defeasible fees i. Mahrenholz v. County Board of School Trustees 417 N.E.2d 138 (App. Ct. IL 1981) [p. 208] (1) FACTS:

33

ii.

that says: this land to be used for school purpose only: (2) HELD: (a) Resolution of case depended on construction of deeds language (b) The deed created a fee simple determinable with the possibility of reverter [A] The use of the phrase for school purpose only suggested that O wanted to give the land to the school only as long as it was needed by the school and no longer (c) The phrase otherwise to revert to the grantors herein suggests that the occurrence of the event would trigger a mandatory return rather than a permissive return Total restraints on alienation of a fee simple interest: void (1) E.G.: to A and his heirs, provided that A cannot transfer his interest in the land to any other person. A has fee simple absolute. (2) If the restrictions are voided, then the conveyance becomes a fee simple absolute (3) Why void? (a) Voiding promotes more efficient use of the property Partial restrictions on alienation of a fee interest (1) I.E., restrictions that only prohibit transfer to some people and/or for limited duration: (2) E.G.: O conveys one of those houses to A and his heirs, so long as he does not sell, devise, or otherwise transfer the property to undergraduate students in the next 20 years (a) This restriction would be valid if O lives next door and is sensitive to noise due to her heart disease (3) Enforceability (a) Courts split (b) Restatement of Prop: restrictions are valid if a reasonable purpose exists, the duration is limited, and the scope is limited Restrictions on land use (as opposed to alienation): restrictions are valid (1) Generally, restrictions on the use of property are permitted if they are not so onerous as to affect marketability of the property (2) What about restrictions not only on how the property should be used but also who can use it? Jurisdictions split. Courts that have upheld such restrictions have interpreted the restrictions as meaning to limit the use of the property to the purpose for which the grantee was created (example: to the city for its own city purposes only) (3) Onerous restrictions on use that materially and adversely affect marketability of the property are void

(a) O conveyed an acre of their farm to (school district) with a deed

iii.

iv.

(a) E.G.: O has a house in Cincy and wants to convey it to his son A,
but does not want his son to sell it to anyone else. He knows his son loves swimming outdoor in winter, so O conveys to A so long as the swimming pool in the yard is used for swimming at least 300 days in any given year. (This restriction will probably be invalid due its onerous nature in regards to the marketability of the property)

34

(b) E.G.: Falls City v. Missouri Pacific Tailway Co., Note 1, p. 220: as
long as the land was used as a site for the railroad companys divisional headquarters

v.

[A] Falls City, NE, conveyed the land to the MO Pacific Railway Co as long as the land was used as a site for the railroad companys divisional HQ, and in case it should be abandoned for such use, the land was to revert back to the city [B] Then, Railway Co. moved its divisional HQ away from Falls City, NE [C] Railway Co. filed an action to quiet title against the city [D] HELD: the restriction was invalidated b/c it unreasonably affected the marketability of the land adversely by (essentially) completely restricting alienation. [E] Bai likes this better than the Mountain Brow Lodge view but organizations are more lenient when the grantee is a charitable organization Restrictions on who can use the property (1) Courts split (2) E.G. Mountain Brow Lodge (a) FACTS: [A] Appellants are a non-profit org. who recd real prop from Toscanos [B] Respondents are trustees and admins of the estates of the deceased grantors [C] Conveyance was a deed which read as follows: (i.) Said property is restricted for the use and benefit of the second party [Non-profit Org], only; and in the event of the fails to be used by second party or in the event of sale or transfer by the second party of all or party of said lot, the same is to revert to the first parties herein [Grantors], their successors, heirs or assigns (ii.) I.E. if non-profit org fails to use the real prop or sells/transfers part/all of the real prop, then the real prop reverts to the trustees & admins of the deceased grantors estate (b) POSTURE: [A] Appellant fraternal lodge sought review of a judgment from the Superior Court of Merced County (California), which refused to quiet title to property gifted to appellant by certain grantors, of whose estate respondents were trustees and administrators. (c) OVERVIEW: [A] Property at issue was gifted to appellant fraternal order, but the deed restricted the use and benefit of the property to appellant only. (d) RESULT: [A] Affd the judgment for respondents, trustees and administrators of the grantors' estates, (e) RATIO: [A] Restraint on alienation contained in the deed was severable from the restriction on use and did not render the use restriction invalid [B] Turning to the use restriction, the court held that it created an enforceable condition subsequent on the estate granted, with

35

title to revert to respondents if the land ceased to be used for lodge, fraternal, and the similar purposes for which appellant was formed. [C] The court held that although the condition had the effect of destroying the alienability of the land, it was not invalid. [D] The court noted that the distinction between a covenant which restrained the alienation of a fee simple absolute and a condition that restricted land use and created a defeasible estate was long recognized at common law and remained recognized in California. (f) HELD: [A] The restriction on use contained in a gift deed to appellant fraternal lodge was a valid and enforceable condition subsequent even though it was tantamount to a restrain on alienation (g) BAI: [A] Majoritys view does not make any sense b/c parties can easily circumvent the ban on alienability restrictions by creating a restriction on who can use the property that essentially functions the same as a restriction on alienability [B] Courts are more willing to uphold a restriction on the use of real property if the property is given to a charitable organization and is restricted to use by that charitable organization d. Restrictions on alienation of life estate i. Courts are generally willing to uphold restrictions in this area ii. Why? (1) The issue of waste is not as prevalent b/c the life estate interest is going to terminate upon the grantees death anyway (2) Also, a grantee of a life estate will probably greatly struggle to convey the real property anyway; so, addtl restrictions arent really going to affect the situation that significantly iii. For Legal Life Estates (1) Promissory Restraints (a) Usually upheld b/c A has explicitly promised not to transfer (2) Forfeiture Restraints (a) E.G. O conveys to A for life, but if A seeks to transfer her interest to anyone, her interest ends and the prop goes to B (b) Usually upheld by courts (3) Disabling Restraints (a) E.G. O grants land to A for life, but has no power of transferring this life estate to any other person (b) Restrictions are usually voided by courts (c) Voided b/c the prop cannot be transferred to anyone under a disabling restraint iv. For Equitable Life Estate (1) Fee simple interest of a trustee (who may or may not be the beneficiary) with the remainder going to 3rd P (2) Restrictions are upheld, including disabling restrictions (3) E.G. O grants $10M in trust to pay the income from the fund to my son during his life but my son has no power to transfer any interest in such income. (spend thrift trust) (4) Rationale for upholding the disabling restrictions

36

(a) Grantor has a legit interest in providing the son a steady income during his life that is free from the reach of his creditors as a result of his irresponsible financial management (b) I.E. Protect the son from his own stupidity F. Practice Problems 1. O conveys to A and his heirs, but if A dies, to B and his heirs. What estate does A hold? a. The language used reflects a fee simple subject to executory limitation, but the limitation is very strange b. It is a life estate in A 2. O conveys to A and his heirs so long as the property is used as As residence. A holds what estate? a. Fee simple determinable in A, OR b. Determinable life estate (A cant live there after he dies) c. Fee simple absolute to A 3. O conveys to A and her heirs, on the express condition that the property be used only for residential purposes, but if it ceases to be used for such purposes, then O and her heirs shall have the right to reenter. What estate does A hold? a. Fee simple subject to condition subsequent 4. O conveys to A and his heirs, it being my wish and purposes in making this conveyance that the property be used for residential purposes. Whats As estate? a. Fee simple absolute 5. O writes to A: I give my house and lot to you for your residence. Dont sell it. Let your sister have the rest of my property. What is the state of estates as a result? a. NOT a fee simple absolute b/c it says, Dont sell it and for your residence. b. It can be anything: fee simple determinable, determinable life estate, fee simply subject to executory limitation, life estate 6. To A so long as he refrains from smoking a. Fee simple determinable 7. To A so long as he refrains from smoking, otherwise to B and his heirs a. Fee simple subject to executory limitation 8. To A, but if A shall ever smoke again, Grantor has the right to repossess the property a. Fee simple subject to condition subsequent 9. To A, but if A shall ever smoke again, to B and his heirs a. Fee simple subject to executory limitation. i. The interest doesnt go back to O, it goes to B. ii. B in this example holds an interest for executory interest. G. EXAMPLES: 1. Fee simple absolute: to A (and his heirs) 2. Fee tail: to A and the heirs of his body; to A and his heirs and if A dies without heirs, to B and his heirs. 3. Life estate: To A for life. 4. Fee simple determinable: To A so long as A doesnt have a dog 5. Fee simple subject to condition subsequent: To A, on that condition that A doesnt get a dog 6. Fee simple subject to executory limitation: To A but if A shall get a dog, to B. 7. Leasehold interest: To A for 5 years. 8. Determinable life estate: To A for life, unless he takes up smoking again. 9. Determinable leasehold interest: To A for 5 years, so long as he only has 1 roommate III. Future Interests

37

Present Interest Fee simple absolute Defeasible Fee - Fee Simple Determinable - Fee Simple Subject to Cond. Subsq - Fee Simple Subject to Executory Limitation Life Estate Lease Assignment of Lease Sub-Lease

Name of Future Interests

Holder of Future Interest

Possibility of Reverter Right of Entry Executory Interest Reversion Remainder Reversion Reversion Reversion

Grantor Grantor 3rd person Grantor 3rd person Lessor Landlord Sub-Lessor

A. Definition 1. Legal interests in property that are not currently possessory but either will or may become possessory in the future 2. Future refers to possession 3. E.G. O grants land to A for life, remainder to B a. As interest is present b. Bs interest is future 4. A person vested with a future interest (B) has legal rights and liabilities in the present a. The person can sell or give away his interest b. Person can sue to enjoin the individual (with present interest) from committing waste c. Person can sue 3rd P who are injuring the land or claiming the title adversely d. Although a future interest does not entitle its owner to present possession, it is a presently existing interest that may become possessory in the future B. 5 Types of Future Interests 1. Reversion 2. Possibility of Reverter 3. Right of Entry 4. Remainder a. Vested Remainder i. Indefeasibly vested remainder ii. Vested remainder subject to partial divestment (or subject to open) iii. Vested remainder subject to complete divestment b. Contingent Remainder c. Vested Remainder, subject to partial or complete divestment 5. Executory Interest a. Springing executory interest b. Shifting executory interest C. Reversion 1. If O, a fee simple owner, granted the land to A for life, the land would revert back to O at As death. Os right to future possession is called a reversion. If O dies during As life, Os reversion passes under his will or to his heirs, and at As death whoever owns the reversion is entitled to possession of the land. 2. Characteristics a. A retained interest, which remain bested in the transferor. b. Freely transferable i. By alienation, ii. In will, iii. By inheritance (if the holder dies w/o will) c. Not subject to Rule against Perpetuities d. Future interest is vested if it is:

38

3.

4.

5.

6.

7.

i. Created in a known person, AND ii. W/o a condition precedent attached e. Grantor conveys a lesser estate than what he owned. f. All reversions are retained interests, which remain vested in the transferor Note: a. Once the lesser estate expires, the remaining interest goes back to the grantor. Grantor holds the reversion. b. O does not have to own fee simple in order to create a reversion c. Hierarchy of Estates i. Fee simple absolute Fee Tail Life Estate Leasehold Estate Example 1: a. O conveys to A for life, remainder to B if he graduates from law school b. O holds a reversion. c. What if B graduates from law school? i. The definition of future interest is an interest that will or may be given. Therefore, O still has a future interest/reversion even if it is a contingent on graduating. The reversion disappears when he graduates from last school. We look at the time of conveyance. Example 2: a. O owns fee simple absolute and conveys to A for life, remainder to B. b. O does not hold a reversion. c. What happens if B dies before A? i. Bs estate would get the property because it is a fee simple absolute. Example 3: a. O conveys to A for life. A leased it out to B for as long as you wish to live there. b. Both A and O hold a reversion When a reversion is retained, it may or may not be certain to become possessory in the future. E.G. O conveys to A for life, then to B and her heirs if B survives A a. If B dies before A, O will be entitled to possession at As death, by reversion b. If A dies before B, Os reversionary interest is divested at As death and will never become possessory c. NOTE: Do not call Os interest (in this or similar examples) as a possibility of reversion. That terminology is wrong and can be confused w. a possibility of reverter

D. Possibility of reverter 1. Associated present interest: (usually) fee simple determinable a. Though (technically) a life estate holder can create a possibility of reverter by conveying his life estate to another, determinable on the happening of an event i. E.G. O conveys to A for life or until he no longer wishes to live there 2. Characteristics a. Vested at creation in grantor i. May endure a long time and possibly forever b. Not subject to Rule against Perpetuities c. Transferability: i. Common law: (1) Not alienable or devisable in will b/c it is a mere possibility and as such it is not recognized as a transferable estate (2) Can be inherited if holder dies w/o will ii. Modern view (most jurisdictions): yes (1) Alienable, devisable in will, and inheritable d. A possibility of reverter is a future interest remaining in the transferor or his heirs when a fee simple determinable is created 3. Example 1:

39

a. O conveys to A so long as A uses it as a bakery i. Determinable life estate. b. O has a possibility of reverter E. Right of entry 1. Associated present interest: fee simple subject to condition subsequent. 2. Transferability: same as possibility of reverter. a. Common law: no b/c it is a mere possibility b. Modern view (most jurisdictions): yes 3. E.G. O grants to A, provided that A shall refrain from smoking, in which case O shall have the right to terminate As interest 4. Characteristics a. Vested at creation in grantor i. May endure a long time and possibly forever b. Transferability (same as possibility of reverter) i. Common Law: (1) Not alienable or devisable in will b/c it is a mere possibility and as such it is not recognized as a transferable estate (2) Can be inherited if holder dies w/o will ii. Modern view (most jurisdictions): yes (1) Alienable, devisable in will, and inheritable (2) SO in 3, O can transfer to A c. Not subject to Rule against Perpetuities F. Remainder 1. Definition a. A future interest in a grantee (not grantor) that will become possessory or is capable of becoming possessory upon the natural expiration of the preceding interest in the prop b. Elements of Definition i. A future interest is in a grantee; not by the grantor ii. Becomes possessory or is capable of becoming possessory ONLY upon the natural expiration of the present interest (1) E.G. O grants land to A for life, then to B and his heirs (a) A holds a life estate and his death marks the time when B has possessory interest. [A] His interest was not cut short. (b) As death is a natural termination of As interest (c) Thus, Bs interest is a remainder (2) E.G. O grants land to A for life, but if A smokes, the prop goes to B (a) A holds a life estate. Death is the natural expiration of a life estate. (b) A smoking is not a natural termination of As interest (c) Thus, Bs interest is not a remainder. It is an executory interest. (3) O grants to A for life (a) No remainder because the future interest is in the grantor. O holds a reversion iii. Capable of becoming possessory is sufficient (1) E.G. O grants land to A for life, then to B if B becomes a lawyer, else to C (a) Bs interest is a remainder b/c it is capable of becoming possessory (b) Cs interest is an executory interest (2) as long as there is a possibility that this interest will become possessory upon the natural expiration of the preceding estate, it satisfies the

40

definition, even if the future interest holder never actually gets possession of the property. 2. Remainder can be in any form of estate a. Fee simple absolute, a life estate, or a term of years b. Example: O grants land to A for life, then to B for 5 years, then to C for life, then to D and his heirs. i. B holds a remainder in term of years ii. Cs remainder is a life estate iii. Ds remainder is in fee simple absolute 3. Vested remainders a. 2 Requirements for Vested Remainder i. Grantee is a known/ascertainable person, AND ii. No condition precedent is attached to grantees interest (1) Other than the natural termination of the preceding estates. b. Example 1: O conveys to A for life, then to B and his heirs i. Bs remainder is vested c. Example 2: O conveys to A for life, then to B and his heirs if B survives A i. Bs remainder is contingent upon B surviving A ii. Surviving A is referred to as the condition precedent (1) If B dies before A, property goes back to O (2) If this restriction was not there, and B died before A, it would go back to O. iii. Therefore, this is not a vested remainder d. Example 3: O conveys to A for life, remainder to Bs children i. At the time of the grant, B is unmarried and has no children. ii. B holds no interest iii. Bs children hold a contingent remainder, NOT a vested remainder iv. If B dies w/o kids, then it reverts to O v. If A dies before B has kids, then it reverts to O. This reversion may be temporary b/c if B subsequently has kids, then Bs kids can claim the interest

e. Indefeasibly vested remainder


i. ii. iii.

Remainder is certain of becoming possessory and cannot be destroyed or divested by subsequent occurrence of events E.G. O conveys to A for life, then to B and her heirs (1) B has an indefeasibly vested remainder in fee simple absolute (2) Nothing that happens in the future could divest Bs interest. It is his. E.G. O conveys to A for life, then to B for life, then to C and his heirs (1) B has an indefeasibly vested remainder in life estate (nothing that happens in the future can cut short the interest=indefeasibly vested) Even if B dies before A, legally speaking his interest is not divested it just legally expires. Therefore, it is an indefeasible interst. (2) C has an indefeasibly vested remainder in fee simple absolute

f. Vested remainder subject to open i. A.K.A. vested remainder subject to partial divestment ii. Definition: A vested remainder created in a class of grantees (1) w/o condition precedent attached; (2) at least one member of the class exists at the time of the grant (3) Note: the first 2 conditions ensure that the remainder is vested

41

iii.

iv.

(4) **the class size may increase after the time of transfer E.G. O conveys to A for life, then As children and their heirs (1) If A, at time of conveyance, has one child B, then the remainder is vested in B subject to open to let in later-born children of A (2) Bs exact share cannot be known until A dies (3) If A has no child at the time of conveyance, the remainder is contingent Characteristics: (1) Vested remainder subject to open cannot be completely divested upon death of existing interest (a) E.G. in 1990, O conveyed to A for life, then to Bs kids. At the time of conveyance, B had one child, C1. However, C1 died in 1991, while A was still alive. In 1992, Bs other child C2 was born. In 1993, A died. (b) Issue What is the state of estates upon As death (c) Answer [A] C1s interest was vested in 1990, and this interest cannot be completely divested upon As death [B] Thus, C1s estate and C2 share the property (2) Freely transferable by alienation, will, and inheritance (a) E.G. O grants to A for life, then to Bs kids. At the time of the grant, B has one child, C1. 20 years later, C1 dies survived by wife. Both A and B are still alive. (b) Issue Does wife have any interest in the property? (c) Answer Yes. Wife inherits C1s vested remainder interest. (3) Rules for Closing the Class in a vested remainder subject to open: (a) When it is physically impossible to have new entrants to the class. [A] Note: any living person is deemed capable of having children. Therefore if someone is 80 you cannot close the class (b) Rule of Convenience. [A] Class is closed if any member of the class is entitled to immediate possession in absence of evidence of contrary intent of the grantor [B] Example 1: O conveys to A for life, then to Bs children. (i.) At the time of the conveyance, B has one child C1 (ii.) C1 holds a vested remainder subject to open. (iii.) At As death, C1 is entitled to immediate possession. (iv.) Thus, Rule of Convenience CLOSES the class, due to C1s entitlement to immediate possession and despite the possibility that B could still have more children (v.) C1 holds fee simple absolute. [C] Example 2: O conveyed to A for life, then to Bs children. (i.) At the time of conveyance, B has one child C1 (ii.) But, after conveyance, C1 died while A and B were still alive (iii.) Then, A dies, survived by B who has no living children (iv.) What is the state of the estates upon As death? (v.) Answer: C1 holds a vested remainder subject to open. C1s interest cannot be taken away but is subject to partial divestment. At As death, C1s estate will be entitled to possession. Thus, the Rule of Convenience closes the class at As death. Cs estate has the entire property

42

g. Vested remainder subject to complete divestment i. Vested remainder has a condition subsequent attached, the occurrence of which completely divests the remainderman of his interest ii. Example: O conveys to A for life, then to B and his heirs, but if B does not complete law school within 10 yrs of this grant, then to C and his heirs. (1) B holds a vested remainder subject to complete divestment (2) Completing law school w/i 10 yrs of grant is the condition subsequent, the non-occurrence of which can completely divest B of his vested interest iii. Transferability (1) Transferable b/c it is vested (2) Transferee holds the same interest as transferor (3) E.G. O conveys to A for life, then to B and his heirs, but if B does not have a law degree at As death, to C and his heirs Then, B conveys to D and his heirs. What estate does D hold? (a) D holds a vested remainder subject to complete divestment (b) If B has not recd a law degree when A dies, then D gets nothing (4) E.G. O conveys to A for life, then to B and his heirs, but if B does not survive A, to C and his heirs. B dies before A and Bs will says, I give everything to my son F. What estate does F hold? (a) F gets nothing b/c he is f****d.

h. Note: vested remainder can be subject to open and to complete divestment i. Example: O conveys to A for life, then to Bs children, but if B dies before
A, to D and his heirs. At this conveyance, B has one child C1 (1) What estate does C1 hold? (2) Answer: C1 holds a vested remainder subject to open b/c B may have more kids, and C1 holds a vested remainder subject to complete divestment b/c the condition subsequent (B possibly dying before A) can completely divest C1 of his interest. i. Transferability of vested remainder: freely transferable.

4. Contingent remainder a. A remainder is contingent if: i. It is given to an unascertained person, OR ii. It is made subject to a condition precedent (1) Condition precedent CANNOT be the natural termination of the preceding estates iii. **In either situation, the remainder is not currently ready to become immediately possessory upon the expiration of the preceding estate b. Example 1: O conveys to A for life, then to the heirs of B i. Remainder is contingent b/c it is given to an unascertained person ii. Heirs of B cannot be unascertained until Bs death, even if B has children iii. No living person has heirs, only heirs apparent iv. If Bs heirs apparent do not survive B, they will not Bs heirs v. The words heirs of B refer only to persons who survive B and are designated as Bs intestate successors by the applicable statute of intestate succession c. Example 2: O conveys to A for life, then to B and her heirs if B survives A. i. Remainder is contingent b/c the language if B survives A subjects Bs remainder to a condition precedent ii. B can take possession only if B survives A

43

d. Example 3: O conveys to A for life, then to B and her heirs if B survives A, and if B does not survive A to C and his heirs. i. This situation creates alternative contingent remainders in B and C ii. Bs contingent remainder is subject to the condition precedent of surviving A iii. Cs contingent remainder is subject to condition precedent of B not surviving A e. Condition precedent v. condition subsequent i. Generally (1) Condition precedent conditional conveyance. There is no grant if the pre-condition is not satisfied. (2) Condition Subsequent unconditional conveyance. No pre-condition to satisfy before the possessory interest can become effective, but it can be divested (3) Look for the commas! ii. E.G. O conveys to A for life, then to B and her heirs if B survives A, and if B does not survive A, to C and his heirs. (1) This gift doesnt even exist if B does not survive A. (a) This condition must be satisfied before the possessory interest exists. (2) B contingent remainder (3) C alternative contingent remainder iii. E.G. O conveys to A for life, then to B and her heirs, but if B does not survive A, to C and her heirs. (1) There is no precondition attached to that conveyance. It is a complete grant. (a) This gift can be taken away later. It is a complete grant now, but it can be taken away later. (2) B vested remainder subject to divestment (3) C executory interest iv. E.G. O grants to A for life, then to B if B gives up smoking (1) B contingent remainder (2) O reversion v. E.G. O grants to A for life, then to B, but if B does not give up smoking, then to C and his heirs. (1) B vested remainder subject to divestment (2) C executory interest f. Distinguish condition precedent from precatory language i. E.G.: O grants to A for life, and in the event of As death, to B and his heirs. ii. There is no condition precedent to Bs interest iii. Precatory language language in a will that reflects the testators wish/desire g. Alternative contingent remainders i. E.G.: O conveys to A for life, then to B and her heirs if B survives A, and if B does not survive A, to C and his heirs. B and C hold alternative contingent remainders h. Transferability of Contingent Remainder i. ii. iii. iv. v. Freely alienable, devisable, and inheritable The transfer is subject to all conditions attached to the interest in the original grant ( E.G.: O grants to A for life, then to B and his heirs if B survives A. B dies before A and in his will B says, to C, my interest in this property. C gets nothing

44

i.

Contingent remainders Destructibility i. Note common law does not like contingent remainders and seeks ways to destroy them whenever possible. ii. Trusts can be used to avoid destruction of contingent remainder iii. It is unvested at or before the natural termination of the preceding estate (1) Example: O conveys to A for life, then to As children, if the age of 21. (a) Two years later, A dies leaving two children C (age 8) and D (age 4). (b) The interest in As children is destroyed b/c their interest fails to vest at or before the termination of the preceding As death. [A] The preceding life estate has come to its natural end, but this contingency in the remainder interest is still there. [B] It is not resolved [C] At this point, the law says that because this contingency is not yet satisfied, this contingent remainder in As children is destroyed. [D] The property goes back to O in the form of a fee simple absolute. (2) Modern Modification (a) Note: Majority (ab ) of US jurisdictions have abolished this rule. (b) In those jurisdictions, at time of conveyance (from example above): [A] A holds life estate [B] As kids hold contingent remainder [C] O holds reversion (in case contingency is not satisfied) (c) In those jurisdictions, at As death (from example above): [A] O gets property back in fee simple subject to executory limitation (i.) Only gets it temporarily until the contingent remainder is satisfied. (ii.) There are still some minority jurisdictions that still follow the common law. [B] C and D have a springing executory interest, which would divest Os possessory interest if either or both C & D reach age 21 at some point (i.) C and/or D claim the property in fee simple absolute (ii.) If neither reach 21, then O has fee simple absolute iv. By merger of two consecutive vested interests (1) Example: O conveys to A for life, then to the first son of A, who reaches 21. (a) At the time of the grant, A has one son who is 15. (b) Subsequently, O conveys to A all my residual interest in the property (c) What is the state of estate upon the first and second conveyance? [A] A has a life estate and his son has a contingent remainder (condition precedent and it is given to an unknown person because we dont have anyone at the time that is 21) O would have a reversion upon the first conveyance. (d) A now holds a vested life estate and the next vested interest Os reversion (e) The two interests merge into fee simple absolute. (f) The contingent remainder in As children is destroyed (g) What happens if there is an intervening contingent remainder lying between 2 vested interests? (h) What if before Os second conveyance to A, As son reaches 21

45

[A] O has nothing left. [B] Once the son turned 21, his interest is vested and O cant try to convey anything else to A. (i) To A for life, then to As eldest child B, but if B fails to reach age 21, then the property goes back to O and his heirs [A] One year later, B is only 15 years old. O makes the second conveyance of all my residual interest in the property to A. [B] After the first conveyance (i.) A holds a life estate (ii.) B has a vested remainder subject to complete divestment (condition subsequent. LOOK AT COMMAS) (iii.) O holds a reversionary interest [C] After the second conveyance (i.) A holds a life estate (vested interest) as well as Os reversion through the conveyance (ii.) But there is another intervening interest that is held by someone else. (iii.) The merger doctrine cant work to destroy a vested remainder subject to divestment. (2) EXCEPT: (a) Two consecutively vested interests were granted in the same instrument (b) E.G: O to A for life, then to As kid for life, then to A and As heirs (c) A has no children at the time of the grant. (d) Although A holds vested estates both before and after contingent remainder, those interests cannot be merged to destroy the contingent remainder of As children b/c As interests were granted in a single conveyance. (3) The merger doctrine has been abolished in many states, but some still continue to follow it. By renunciation of the preceding life estate (1) Since contingent remainder depended on the existence of the preceding life estate, it is destroyed when the holder of the preceding life estate renounces his interest. (2) E.G. O conveys to A for life, then if A marries, to As spouse and spouses heirs, but if A dies w/o marrying, to the Red Cross. (a) At the time of conveyance, what is the state of estates? [A] A life estate [B] As spouse contingent remainder [C] Red Cross alternative contingent remainder [D] O reversion (b) Then, after the conveyance, A (who is still single) renounces her life estate [A] When A renounces her interest, then all the contingent remainder interests are destroyed and the property reverts to O (3) Modern Statutory Modification (a) Property goes back to grantor in the form of fee simple subject to executory limitation, until the death of the life estate holder (b) Thus, in example above, property would revert to O, who holds possession until A died (when it would pass to Red Cross or As spouse, if A married)

v.

46

(c) E.G. O conveys the house to my daughter D for life, remainder to my grandson GS if he reaches age 21, otherwise to my granddaughter GD. [A] Then, D renounces her life estate. [B] Prop reverts to O, until D dies G. Executory interest(shifting and springing) 1. Definition: a future interest in a transferee that must, in order to become possessor: a. Divest or cut short some interest in another transferee (this is known as a shifting executory interest), or b. Divest the transferor in the future (this is known as a springing executory interest) 2. Shifting executory interest a. Divests the future interest of a grantee (as opposed to the grantor) before it comes to a natural expiration. i. Different from a remainder. A remainder never cuts off a prior interest, but merely awaits the prior interests natural termination. An executor interest, by contrast, divests or cuts off a prior interest before the latters natural termination. (1) O bargains and sells to A for life, then to B and his heirs if B survives A, otherwise to C and his heirs. B and C each have contingent remainders. (2) O sells to A for life, then to B and his heirs, but if B should die before A, to C and his heirs. Here Bs interest is vested subject to divestment because the but if.. divesting language comes in a separate clause, and Cs interest is therefore an executory interest. b. Associated preceding interests: i. Fee simple subject to executory limitation ii. Life estate subject to divestment (example: to A for life, then to C (Bs child) for life, but if B does not survive A, then to D and his heirs. Ds interest is shifting executory interest that divests Cs life estate if B dies before A). 3. Springing executory interest a. Divests the interest of the grantor (example: to A for life, then to B if B gives A a proper funeral. i. Bais favorite ii. A has a life estate iii. O has a vested reversionary interest iv. B holds a springing executory interest). (1) His interest doesnt become possessory until the funeral happens (a) During the time gap between when A dies and the funeral, O has the property [A] If later on B arranges a proper funeral, his interest will become possessory, but only when he cuts short Os interest. b. I hereby give my property to the person who restores my vision i. Grantor is blind. ii. This person who is going to restore vision in the grantor naturally holds a future interest. (1) Not presently possessor because he hasnt yet restoring the vision in the grantor. (2) This future interest becomes possessory upon the natural termination of the grantors possessory interest (his death), but when he restores the vision (a) The grantors possessory interest is cut short.

47

4. Note: Executory interests become possessory automatically upon occurrence of


divesting events.

5. O conveys land to A, but if A stops using the property for school purposes at any time

during the next 20 years, then to B and his heirs. a. Starting from the 5th year, A stopped using the property for school purposes. B did nothing until the end of the 20th year. When A claimed adverse possession against B, what is the result? i. The statute of limitation is 10 years. ii. A had a fee simple subject to executor limitation (1) At year 5, the property was given to B (shifting executory interest) (2) A continued to use this property adversely throughout the entire limitation period. (a) He acquired title through adverse possession. (b) Make sure when an interest because possessory automatically, the failure to come to the property right away may give rights to claims by adverse possession.

H. The Modern Trust 1. Intro a. Trustee holds legal title to the trust property and manages it for beneficiaries benefit b. Beneficiaries hold equitable interests in the trust property c. Rule against restraints on alienation do not apply to beneficiaries equitable interests d. Spendthrift Trusts i. Settlors can protect the beneficiaries interests by making them inalienable ii. Such trusts can be drafted to prevent beneficiaries from transferring or borrowing against the trust and to exclude creditors from using trusts to collect on debts e. Dynasty (or Perpetual) Trusts i. Some STs allow individuals to create trusts that control wealth forever into the future 2. Example: a. O conveys to X in trust to pay the income to A for life, and then to pay the principal to As children who survive A. i. X has express power to sell the trust property (or re-invest it) ii. X has the legal fee simple iii. A has an equitable life estate, entitling A to all the income that the trust generates iv. As children have an equitable contingent remainder v. O has an equitable reversion vi. Upon As death, X conveys trust property to As kids (if any survive A) or to O I. Special rules to remember 1. The Rule in Shellys Case2 a. If (1) one instrument (2) creates a life estate3 in real property and (3) a remainder in the life estate holders heirs (or heirs of his body), and (4) the life estate and the remainder are both legal or equitable, the remainder becomes a remainder in the holder of the life estate
2

Most US jurisdictions have abolished the rule, but there are 3 or 4 that still apply this rule. In addition, this rule applies (in jurisdictions that have abolished this rule) for grants made prior to the abolishment. In many such jurisdictions, the Rule was abolished in recent years, so litigation is still expected around this Rule. 3 Or a fee tail if fee tail is recognized, given the abolishment of fee tail in most jurisdictions, the applicability of this rule is further limited to the situation where the present possessory interest is a life estate, with remainder to the heirs of the life estate holder.

48

b. Example: to A for life, then to As heirs.


i.

Without applying the Rule in Shelleys case, As heirs hold a contingent remainder because the heirs are unknown people at the time of the grant. ii. The Rule in Shelleys case treats the contingent remainder in As heirs as a vested remainder in A.Since A holds both the life estate and the next vested remainder, by merger rule, A now holds a fee simple absolute, and the land is immediately alienable c. Elements of the rule: i. Life estate and remainder were created in one instrument. ii. Heirs interest must be a remainder as opposed to executory interest (example: O conveys to A for life and the, one day after her death, to her heirs. Shellys rule does not apply because As heirs hold a springing executory interest). iii. Life estate interest and the remainder must be both legal or equitable (example: O conveys To A for life, and then in trust to her heirs. The rule does not apply). iv. Heirs must not refer to specific persons (example: O conveys to A for life, then to As heirs who are As children. The Rule does not apply). d. ABOLISHED nearly every jurisdiction has eliminated the Rule in Shelleys Case i. But, the abolishing is still fairly recent in some STs and does not apply retroactively ii. Thus, it is still useful to be familiar with the Rule in Shelleys Case iii. Only 3-4 STs have not abolished this rule e. NO EXAM QUESTIONS on the RULE in SHELLEYs CASE 2. Doctrine of worthier title a. If any inter vivos conveyance (rather than a conveyance by will) has created a future interest in the heirs of the grantor, the future interest is void b. Example: O conveys inter vivos to A for life, then to Os heirs. i. Future interest is created in Os unascertained heirs (in the form of a remainder). ii. This future interest is void. iii. Thus, O has a reversion c. Most states still apply this doctrine to inter vivos conveyances. However, it is now only a rebuttable presumption that the grantor does not intend to create any future interest in his heirs in an inter vivos conveyance. d. Note: heir must not refer to any specific person. IV. The Rule Against Perpetuities (RAP)

A. Common law RAP


1. Introduction a. No interest is good unless it must vest, or fail to vest, not later than 21 yrs after some life, in being at the creation of the interest, ends i. At the creation of the interest refers: (1) to moment of testators death in will transfer, (2) OR, to time of conveyance in an irrevocable inter vivos transfer ii. RAP does not apply to a revocable inter vivos transfer b. To defeat the RAP, one must prove that a contingent interest is certain to vest or terminate no later than 21yrs after the death of some person alive at the creation of the interest c. To A, so long as property is used for residential purposes, otherwise to X i. The date is March 19, 2010. In the next 200 years, the property will be used as a residential property.

49

However, one of As decedents started using the property for commercial properties in the year 2211. (1) The property goes back to X. X is no longer available so it goes to his decedents (2) Property interests shifts according to the wish of the grantor through the incursion of a contingency of the gift. (a) If you think that 25 years accounts for 1 generation, the grantor can control about 8 generations down the road. (b) The law doesnt like this [A] As decedents have this contingency lingering over their head forever. [B] So the law says that it doesnt like the shifting of property interests based on a contingency for too long [C] The grantor has a legitimate interest in dictating how his property should be disposed of, however, the power of the grantor cant last for such a long period of time to affect the more productive use of the property. d. How do we define current generation? i. Any person who is alive during the conveyance? ii. The conveyance starts the time. (the gift comes into place now). So from now until the end of life of the last person to die among all people that are alive at the time of the conveyance. (1) At the time of the conveyance, A B C D E are alive. Among the 5 of them, there has to be one who is the last to die. (a) Assume the last person to die is E. the duration of the current generation is the end of Es life. (2) Therefore, the grantor is allowed to control the shifting of interests in the property through incursion of contingencies which may take place during the permitted period of time (a) If your contingency occurs during the definition of a generation is okay. But if it occurs afterwards, there is a problem. (b) Therefore, after the death of the last person to die + 21 years. e. To A for so long as Cincinnati has no earthquake, else to B and his heirs. i. Bs interest is a contingent remainder. ii. There is a possibility that the property will be shifted from A to B. (1) Is there a possibility that the shift will occur after the expiration of the expiration of the max allowed period? (a) As long as there is any possibility that it will occur after this time it will be voided. (b) We dont have to know the identity of the last person to die in order to determine whether a gift is valid or not. [A] There are 2 basic approaches to pin down the validity of the gift (i.) If the contingency will surely be resolved one way or the other during the life span + 21 years during one of the peoples lifespan, then we have satisfied the requirement of the rule against perpetuities. (ii.) Think of any scenario, no matter how unlikely, but as long as the possibility exists, that everyone dies but the contingency will be satisfied before the 21 years is up. f. Find a person who will enable you to prove that the contingent interest must vest or fail to vest w/i the life, or at the death, of that person, or w/i 21yrs after that persons death. i. That person, if found: (1) is called the validating life (or measuring life)

ii.

50

(2) must be alive at the time of conveyance (3) BUT, need not be mentioned in the instrument itself ii. If validating life is not located, interest is void unless it must vest or fail w/i 21yrs iii. Under common law RAP, it must be conclusively assumed that a person of any age can have a child, no matter what the persons physical condition, or any contrary evidence g. When evaluating a conveyances validity under the RAP: i. Consider only the information and facts in existence at the time of the conveyance (1) Do NOT consider any events that occur after the conveyance, at all! ii. Consider what could happen (even if far-fetched), rather than would did happen h. If a transfer creates more than one interest subject to the RAP, each interests validity must be individually tested. One interest may be valid, while another is not. Also, the interests can each (theoretically) have a different validating life, or share one validating life 2. Interests that are subject to the RAP a. contingent remainder i. important to distinguish between this and a vested remainder subject to divested b. executory interest i. executory interests are deemed vested interests only at times of actual possession. c. class gifts (a.k.a. vested remainder subject to open) i. Vested remainders subject to open are NOT inherently vested under the RAP ii. For a class gift to be vested under the RAP: (1) the class must be closed (i.e., at the time the class gift is created, each and every member of the class must be in existence and identified), AND (2) all conditions precedent for each and every member of the class must be satisfied w/i the perpetuities period (i.e., in accordance w. the RAP) iii. Wont talk about this in class because it is complicated. Thank you Bai. 3. Interests that are NOT subject to the RAP a. Vested interests i. Present possessory interests are vested interests ii. Vested future interests include: (1) Reversion vested interest in grantor (2) Possibility of reverter vested interest in grantor (3) Right of entry vested interest in grantor (4) Indefeasibly vested remainder (5) Vested remainder subject to complete divestment (6) Vested remainder subject to open (**see below) (a) Even though a vested remainder subject to open is vested, it is still subject to RAP if it is a class gift. b. An interest in charity, following another interest in charity i. Example: O conveys to the American Cancer Society, provided that, if a cure for cancer is found, then to the Society for the Prevention of Blindness. (1) Technically speaking, the second gift violates RAP because we can imagine at the time of conveyance after everyone dies there still wont be a cure. (2) But this exception is carved out to encourage others to still give gifts to charities.

51

ii. iii.

The future interest of Society for the Prevention of Blindness is not subject to RAP The future interest by the second charity is not subject to RAP.

4. Reason for the RAP a. 2 competing concerns i. to prevent the grantor from controlling property indefinitely, which harms marketability and alienability of property ii. on the other hand, grantor has a legitimate interest in controlling how his property is disposed of for some reasonable period of time after his death b. A compromise i. Law set a time limit ii. Life span of anyone alive (at time of conveyance) + 21 years iii. **Other possible candidates for the time limit (1) Wait a fixed period of time (e.g. 50 years) (a) Problem: This is a wait-and-see approach, but the Grantee needs to know the validity of his interest at the time of conveyance (2) Specify a fixed period (e.g. 50 yrs) and determine at the time of the conveyance whether the contingency is resolved w/i such period (a) Problem: many gifts will be voided under this approach (3) Grantor names a specific person as measuring life in the conveyance (a) Assumes that Grantor has sufficient legal knowledge, or $$$ for counsel (b) RAP is more generous 5. How to identify the validating (measuring) life a. If you can point to any particular person who is a life-in-being at the time of the conveyance, within whose remaining life span + 21yrs the contingency associated with the future interest is resolved (either by vesting or failing to vest), then the gift/transfer is valid b. Realistically, such a person is more likely to be among those ppl associated with the gift c. Example: O conveys land to my son A for as long as he refrains from smoking, then to the first of his children who reaches age 21. At the time, A has one son who is 2 years old. i. Anyone who is alive at the time of the gift can potentially be the validating life ii. Yet, validating life, if existent, is almost always among the pple associated w. gift 6. Examples: a. O conveys to A for life, then to B if he is admitted to the bar. i. Step 1: Identify the estates granted (1) A: life estate (2) B: contingent remainder (3) O: reversion ii. Step 2: Identify the estates that are subject to the RAP (1) Subject to RAP Bs contingent remainder (2) NOT subject to RAP As life estate and Os reversion iii. Step 3: List all candidates who can serve as validating life (1) O (2) A (3) B (4) Any one related to them who is a life-in-being at conveyance iv. Step 4: Eliminate those who are not born or conceived at time of conveyance

52

(1) For the exam, unless otherwise indicates, assume that every person names in the grant is alive at the time of the grant v. Step 5: Prioritize the list (1) At the top, place the person who is most likely to be the validating life (a) E.g. B because the contingency is about his passing the bar vi. Step 6: Consider each person on list until finding a validating life or exhausting the list (1) B is a validating life, b/c at Bs death it will be conclusive as to whether B passed the bar, and the contingency will not linger more than 21 yrs after Bs death b. O conveys to A for life, then to B if B attains the age of 30. B is now 2 years old. i. Step 1: (1) A life estate; B contingent remainder; O reversion ii. Step 2: (1) Only Bs contingent remainder is subject to RAP iii. Step 3: (1) O, A, B, and any person associated with them iv. Step 4: (1) Everyone is alive at time of conveyance v. Step 5: (1) B should be at top of list, b/c the contingency directly involves B vi. Step 6: (1) B is a validating life, b/c upon Bs death his age will be certain. If B fails to reach 30 before his death, then the contingent remainder automatically fails to vest. c. O conveys to A for life, then to the first child of A who reaches age 30. At the time of the grant, A has two children C1 (28 yrs) and C2 (27 yrs). i. Step 1: (1) A life estate; (2) As First Child to reach 30 contingent remainders (a) Not necessarily C1 or C2, and thus the party is unascertained (3) O reversion ii. Step 2: (1) Only the contingent remainder is subject to the RAP iii. Step 3: (1) A (2) C1 and C2 (3) O (4) As other possible children iv. Step 4: (1) As other children must be eliminated, b/c not lives-in-being at time of grant v. Step 5: (1) C1 and C2, b/c the contingency is which one, if any, reaches age 30 first (2) A (3) O vi. Step 6: (1) Neither C1 nor C2 can be the validating life (a) Either, or both, could die tomorrow, and the identity of As first child to reach 30 still would be unascertained and might not be clarified w/i 21 yrs of C1/C2 dying (2) O cannot be the validating life (3) A cannot be the validating life (4) Thus, the contingent remainder to As first child is invalid under the RAP

53

vii. Step 7: What now? (1) Following common law: (a) At As death, the unvested contingent remainder is destroyed and the property reverts back to O (2) Modern Approach: (a) At As death, the contingent remainder is not destroyed, but the property reverts to O anyway, for the time being at least [A] O holds fee simple subject to executory limitation [B] As first child to reach age 30 holds springing executory interest (i.) At 30th B-day, the prop goes to him in fee simple absolute d. O conveys to A for life, then to As first grandchild. At time of grant, A has one child C1, but no grandchild. i. Step 1: estates created (1) A life estate (2) As first grandchild contingent remainder (a) **interest conveyed to unascertained person is a contingent remainder (3) O reversion ii. Step 2: estates subject to RAP (1) Contingent remainder held by As first grandchild iii. Step 3: possible candidates (1) A (2) C1 (3) O (4) As first grandchild (5) As other children iv. Step 4: eliminate lives-not-in-being (1) As first grandchild and As other children are not alive at the time of conveyance, and thus must be eliminated from list v. Step 5: prioritize (1) C1 (2) A (3) O vi. Step 6: determine if candidates are acceptable as validating life (1) C1 No (2) A (a) At As death in common law, contingent remainder would be destroyed. Thus, following common law, this interest is valid under the RAP. (b) In modern approach, As death does not destroy the contingent remainder. Thus, under modern approach, this interest is NOT valid under the RAP. [A] If C1 dies without having a child and A has another child. A dies the next day. 25 years later, C2 has a child, thus invalidating the RAP. (3) O No e. O grants in will to A for life, then to Os first grandchild. O is survived by children (C1 and C2) but has no grandchildren i. A has a life estate. Os estate has reversion. First grandchild has a contingent remainder. ii. Contingent remainder is subject to RAP iii. Valid because O cant have any more children that can satisfy the RAP. C1 and C2 alive at the grant.

54

(1) Once either C2 or C1 has a child, the contingency is satisfied. (2) Suppose C1 dies today, survived by C2. (a) 25 years later, C2 has a child. The interest in this example is vested more than 21 years later after Os interest. (b) C1 cant be the validating life. [A] Neither of them independently can be the validating life. The 2 of them together can be the validating life. (i.) Why? Because C1 dies before C2. C1 cant be the validating life because C2 could give birth to another child more than 21 years after C1s death. (ii.) Therefore it will either vest or completely disappear f. O conveys land to Os son for life, then to the sons widow for life, then to the sons first child if he is then living. at the time of the grant, Os son is married but has no children. i. O reversion. Os son-life estate. Os sons widow- contingent remainder in the form of a life estate because we dont know who the widow is because the son is not dead yet. First child has a contingent remainder. (1) Both contingent remainders are subject to RAP ii. Sons widow: valid. (1) If the son is the validating life, at the end of his life, hell either have a widow or not so it would be resolved. iii. Os sons first child: invalid (1) The son divorces the current wife after the conveyance. 18 years later he marries a woman who was not born at the time of the gift. Shortly after the marriage, Sam dies and everyone who was alive at the time of the conveyance dies, survived by the widow and the child. (a) The widow lives for 30 years survived by this child. [A] Thus the childs interest is vested more than 21 years after the death of every life in being at the time of the conveyance. (2) Can we use the widow as the validating life? (a) No; because the widow could be, not has to be, someone who is not alive at the time of the gift. (b) The widows life cant be used as the validating life. g. Rights of first refusal can stay and are transferrable. h. Symphony Space v. Peroda Properties i. Add this info in from supplement ii. Note that it is not what did happen under the transaction but what could have happened iii. Since both parties were wrong about the validity of the contract you would think that the contract could be cancelled out of mutual mistake. iv. Court said RAP could invalidate and override the contract. (1) Strike out the part that violates the rule

7. Special rules to remember


a. If the contingency is such that it is unreasonably difficult to verify whether the contingency is resolved, then the interest will be invalidated even thought it technically satisfies RAP i. E.G.: O devises to A for life, then to As children who are living 20 years after the deaths of all the people listed in the Manhattan telephone book who are alive at Os death. (1) As children: technically speaking it satisfies RAP. (a) Certain to vest in the children (b) It will be extremely difficult to verify the death of all individuals in the phonebook

55

(2) But since it is so hard to prove, it will be invalid automatically. b. Contingent life estate is always valid under RAP i. E.G.: O conveys to A for life, then to B for life if one of Bs children is admitted to the bar, then to C. (1) At time of grant, B has children, but none of them has been admitted to the bar. (2) Since B holds only a life estate which ends upon his death regardless whether the contingency is resolved, B can be used as a measuring life for purpose of RAP (a) Although technically speaking Bs interest violates RAP, because the vesting of this remainder can happen after 21 years. [A] However, when this interested is vested, B is already dead and the issue resolved. (i.) Because Bs interest comes to an end naturally upon his death, his interest is valid even though it technically violates RAP. c. Interest certain to vest or destroyed within 21 years of the grant is valid i. E.G.: O conveys To A, but if liquor is served on the property at any time during the next 21 years, then to B. Bs executory interest is valid (1) No matter what, you will have the contingency resolved after 21 years. d. If the interest is certain to vest but more than 21 years from the time of the grant, it is technically invalid under the common law RAP. However, the modern view is that such interest is valid because concern for pre-longed uncertainty is not present i. E.G.: to A 30 years from now (1) Springing executory interest (a) Cuts short the grantors present possessory interest. (2) A is not a validating life. A doesnt have to be alive to take this gift. His estate can take over the gift. e. In states that still follow the common law rule on the destruction of contingent remainder at the natural expiration of the preceding estate, the holder of the preceding estate can be used as a measuring life to validate the contingent remainder for purpose of RAP i. E.G.: to A for life, then to Bs children who reach age 30. (1) If the interest in Bs children is not vested at As death, it is destroyed (2) Thus A can be the measuring life to validate the future interest in Bs children 8. Which clause to strike? a. The condition precedent language or the fee simple determinable language such as so long as, to A if.., while, etc. are deemed a part of the granting language of the first grant and NOT part of the future interest that is invalidated by the Rule. It is not crossed out (example: O conveys to the Red Cross, so long as the land shall be used for purposes of enhancing awareness of autism; and when it shall be diverted from that use, then to A. As interest is invalid. You cross out the part in red font. The Red Cross has a fee simple determinable).

b. Condition subsequent is deemed part of the invalidated future interest. Thus it 56

should be crossed out (example: O conveys to the Red Cross, but if the property

is not used for purposes of enhancing awareness of autism, then to A. As interest is invalid under RAP. You cross out the part in red font. The Red Cross has a fee simple absolute). c. O conveys, to A, so long as the land shall be used for purposes of enhancing awareness of autism, else to B. i. Bs interest is an executory interest and subject to RAP. (1) Bs interest has violated RAP. (2) We need to cross it out ii. Which part of the conveyance language do we want to cross out? (1) Depends on the language used in the conveyance (a) If it uses a fee simple determinable type of language (so long as, while..), this type of language is deemed to indicate the limited duration of the previous gift. [A] To A, so long as the land shall be used for purposes of enhancing awareness of autism. (i.) This language is part of the preceding gift. (ii.) So just crosss out else to B (iii.) Possibility of reverter is not subject to RAP d. To B and his heirs, but if there is an earthquake in Cincinnati, then to C and his heirs. i. Cs interest is an executory interest and violated RAP. (1) There could be an earthquake more than 21 years after everyone alive has died (2) It needs to get out. ii. The condition subsequent type of language used here. (1) This is used to divest the proceeding estate. (2) But look at the part before the comma..this indicates a grant of unlimited duration. It is a complete grant. (3) The earthquake is referring to a condition, if it occurs, that can divest the proceeding interest. (4) This divesting language goes together with the divesting interest. iii. The conveyance should read: to B and his heirs. Fee simple absolute. 9. Corporations a. In most jurisdictions, a corporation cannot be a validating life b. Examples i. To A, so long as Cincinnati remains a Delta hub, otherwise to C (1) C holds executory interest (2) Delta cannot be a validating life (3) Thus, Cs interest violates the RAP B. Major modifications to RAP 1. Uniform Statutory Rule against Perpetuities: a. Most popular approach adopted by about half of the jurisdictions b. This approach fixes the wait-and-see period to a flat 90 years i. Wont determine if the conveyance violates RAP at the time of the conveyance. ii. If the contingency is not resolved at the end of 90 years, then it will be invalidated. (1) To A, so long as Cincinnati has no earthquake, else to be

57

(a) Under traditional common law, Bs interest violates RAP even though the day after the conveyance, Cincinnati has an earthquake. (b) Under the modern approach, Bs interest would be valid because an actual earthquake occurred within 90 years of the conveyance. c. If at the end of 90 years following the creation of the interest, the interest is still in existence and unvested, it is invalid. d. Why 90 years? i. On average, the grantors youngest descendant in-being at the time of the grantors death is 6 years old ii. On average, this 6 year old is expected to live until 75 iii. (75 6) + 21 = 90 years e. Problem with this approach i. Uncertainty during the 90 year period inhibits inalienability of property 2. Other Modifications: a. Note: You are not expected to know other approaches for purpose of the exam. b. Specific modifications to avoid pure technical violations i. Fertility age any woman above 65, or below 13, is deemed incapable of having kids ii. Reduce age contingencies to 21 or below (1) If a gift is to A for life, then to B if he reaches age 24, then to C. (2) 24 is reduced to age 21 3. Qualified abolition of RAP in case of perpetual trust a. The federal estate tax and generation skipping transfer tax i. Applicable to trusts ii. The 3.5million exemption iii. The Exclusion from valuation appreciation until end of trust b. The perpetual trust i. Ex. O dies, and in his will he creates a trust of assets worth 3.5 million with benefits payable to Os child D, remainder to Ds child GS, then to GS children, then to GS grandchildren. c. Congress miscalculation about the status of RAP i. Individual states abolish RAP as applied to trusts to lure wealthy businesses and individuals to the state. Many states have done this.

Nuisance
I. Scope of coverage A. Private nuisance 1. Intentional nuisance (a) You know the way you use your property will interfere. You intend the conduct. This doesnt mean that you like the consequences. i. Chemical company (1) Constant discharge into the public waters. You know of this, but you go ahead and operate it anyways. 2. Unintentional nuisance B. Public nuisance C. Remedies II. Private nuisance Private nuisance occurs when there is interference with private right to use and enjoy land.

58

You use of property interferes with a limited amount of entities. A. Intentional nuisance 1. Elements for cause of action: The conduct is intentional and unreasonable. (a) Intentional: Intentional nuisance occurs when the defendant acts for the purpose of causing interference or knows or is substantially certain that his conduct will cause interference with other peoples use and enjoyment of land. Intentional nuisance can occur when the defendant exercises great care in an effort to avoid it. (b) Unreasonable: There are two main approaches toward determining unreasonableness: i. Threshold test (majority jurisdictions): Under this test, liability arises when interference goes beyond prevailing norm. Examples of circumstances that do not give rise to cause of action include: (1) Unfounded fears arising from the defendants use does not provide cause of action for nuisance. (2) The plaintiffs abnormal sensitivity with regard to his neighbors property use (example, Amphitheaters, Inc. v. Portland Meadows, page 644). (3) Blocking light to a neighbors property (however, cause of action arises if malice is present). (4) Plain ugliness unless spite is the only motive. ii. Balancing the social harm against social utility (minority jurisdictions): Applies in cases where the plaintiff seeks injunction only. There are cases in which the defendants conduct is harmful but the social burden of eliminating the nuisance outweighs its benefit. In such cases, damage should be granted to the plaintiff but the minority approach of determining unreasonableness precludes such outcome (example: Estancias Dallas Corp. v. Schultz, page 646). Note: Courts often lack expertise in evaluating the social harm and social utility. (1) The harm that the conduct has created is balanced against the utility generated by this conduct. (2) If utility exceeds the harm, no nuisance is found. (3) Factors: every factor that is relevant 1. Includes harms/benefits to particular parties to the case a. And harms and benefits to the society at large in theory. 2. In practice, because judges are deprived of mediums of figuring out the utilities or harms to society at large, very often the balancing is limited to the benefits/harms to the parties/neighborhoods involved. (4) Problem: the cement company creates much utility to the neighborhood (employment and other social benefits.) in terms of harm, the property value has depreciated. The court finds that this cement companys utility outweighs the harm. Once the court reaches the conclusion, its bound to find that no nuisance exists. (5) In order to have a cause of action for interference with other peoples interests, you must determine that this conduct is unreasonable. 1. This is why most courts stick with the threshold test. 2. Use this only when determining whether injunction should be granted. (6) The balancing is often biased. (Cement case on page 644)

59

1. Include employment in the neighborhood and the amount it takes to invest in the company. 2. Damage to the property value of the plaintiff 3. However, the court didnt take into account pollution to the air (7) Although in theory it should take into account every harm, it is often biased. 2. Coming to nuisance doctrine The plaintiffs coming to nuisance is a complete bar to his recovery for private nuisance. The nuisance is already there. P lives close to the nuisance place.

3. Intentional nuisance v. trespass (a) Trespass involves an interference with right of possession. It usually requires
physical invasion by a tangible thing.

(b) Nuisance involves interference with use and enjoyment of the property. It does (c)
not involve physical invasion by a tangible thing. Different legal standards for trespass and nuisance: Both are intentional conduct of the defendant and yet the plaintiff does not need to show that trespass was unreasonable.

B. Unintentional nuisance Cause of action for unintentional nuisance requires negligence, recklessness, or abnormally dangerous activity on the part of the defendant. An oil refinery company tried everything possible to avoid spillage. But contrary to their intentions, it went into the water. III. Public Nuisance A. Definition It is interference in public rights rather than private rights of land use. To constitute public nuisance, the nuisance must affect a considerable number of people or an entire community or neighborhood. B. Elements of public nuisance Same as private nuisance, it depends on whether the nuisance is intentional or unintentional. C. The standing to sue Public officials/agencies have the right to sue, but a private citizen can also sue if he has suffered special injury from the pubic nuisance in addition to what other people in the community have suffered (example, Del Webb v. Spur, page 656). IV. Remedies 4 remedies are available. A. Injunction B. Allow nuisance to continue but grant permanent damage to the plaintiff

1. In jurisdictions that use the threshold test in determining whether the defendants
interference with the plaintiffs use of land is unreasonable, courts balance the social

60

harm against social utility in determining whether injunction should be granted (example: Boomer v. Atlantic Cement Co., page 649). C. Injunction as well as awarding damages to plaintiffs

D. Injunction but asking the plaintiff to pay for cost of terminating nuisance under coming to
nuisance doctrine (example: Spur Industries, Inc. v. Del E. Webb Development Co., page 656).

Easement
The right to use someone elses property. I. Scope of coverage (p. 667 701; 709 36) A. Different types of easement B. Creation of easement 1. By grant 2. By estoppel 3. By implication a. implied from prior use b. implied by necessity 4. By prescription C. Transfer and division of easement D. Scope of easement E. Termination of easement II. Different types of easement A. Two ways to classify easements 1. Appurtenant or in gross 2. Affirmative or negative B. Appurtenant v. in gross 1. Appurtenant: benefits the claimants land rather than the person. The benefiting parcel is called the dominant estate; the burdened parcel is called the servient estate. a. If you are the owner of lot 1, and your friend is the owner of lot 2, and you grant him an easement. The granting language says I hereby grant lot 2 the right to go over my property in order to access the public road. b. This easement benefits the property/estate. 2. Easement in gross: Benefits the person who holds the easement. It creates a personal right to use the servient estate (example: a public utility companys right to run a public sewage through the underground of servient estate). a. I grant an easement to my friend who can go over my property in order to access the public road. i. This benefit goes to the owner of lot 2. ii. It benefits a person, not the estate itself. 3. Ambiguous grants: Easement appurtenant is preferred because it is easier to locate the owner of the dominating estate when the servient estate owner wants to negotiate a termination of the easement, and because easement appurtenant increases the aggregate value of the servient estate and the dominate estate. C. Affirmative v. negative easement 1. Affirmative easement: It gives easement holder the right to use the servient estate in specified manners. 2. Negative easement: It confers no right to use the servient estate, but rather a right to prevent the servient estate from being used in specified manners. Note: Most negative easements are appurtenant, but some negative easements are in gross (example: A

61

and B are neighbors and their backyards are divided by a 3-feet tall fence. A is an 80 year old woman with a severe heart disease and thus is especially afraid of noise. At As request, B has agreed not to host a party of more than 10 people in the backyard while A suffers from the heart condition. A holds a negative easement in gross). D. Distinguish easements from licenses The key distinction is that license is revocable at the will of the grantor while easement, once established, cannot be revoked at discretion of the servient estate. A license that by agreement is not revocable is like an easement in essential aspects. If you grant your friend a right to travel across your property to get to the public road, but there is evidence that it is an easement in gross, the transferee to your property will be bound by the pre-existing easement, but if its only a license, then when you transfer your land, the license will cease and the new owner is not bound by that license. 1. Exceptions to a licenses revocability at will of the grantor: a. Estoppel (see our discussion on creation of easement below) b. License with interest (example: a license to across As land to remove timber. A cannot revoke the license until timber has been removed). 2. In case of ambiguity: Courts try to construct parties intent from all circumstantial evidences. Courts prefer license over easement because it is easier to eliminate license. 3. License or easement - difference in consequences: A license binds only the licensor and not his successor in interest; while an easement binds the new owner of the servient estate. III. Creation of easement A. Created by grant, by estoppel, by implication (from prior use or necessity), or by prescription. B. By grant 1. Often via deed, and the most common means of creating easement a. Beware of ST Statute of Fraud requirements 2. Creation of Easement by Reservation a. Reservation in grantor: valid i. Grantor conveys fee simple to the grantee, while reserving easement in himself ii. Common law reqd. grantor to convey fee simple to grantee, who in return conveyed the easement back to the grantor b. Reservation in a 3rd person: i. Majority jurisdictions do not recognize the validity of reservation in a 3rd P (A) To effectuate such reservation, the grantor must take two steps: (1) Convey to the 3rd person in fee simple; (2) 3rd person conveys to the intended grantee in fee simple, but with a reservation of easement in favor of himself ii. Some jurisdictions allow reservation in a 3rd P (A) See Willard v. First Church of Christ, Scientist (1) Owner conveyed land to , but deed explicitly contained easement for church (who was a 3rd P to the deed) (2) Issue: whether court would allow reservation in a 3rd P (3) Result: Court adopted minority view and allowed easement clause iii. Common law reservation in 3rd P is inherently invalid C. Estoppel barring the revocation of a license 1. General Rule: a. A license, the revocation of which is barred by estoppel, turns into an easement b. To bar revocation, licensee must show detrimental change in position due to reasonable reliance upon the license. (Example: Holbrook v. Taylor, page 677).

62

i. There needs to be a license, explicit or implicit. ii. Good faith and reasonable reliance iii. Grantor knows or has reason to know that the grantee will rely on the irrevocability of this license. iv. The commission can be either implicit or explicit. (A) What about mere acquiesce? A and B are neighbors, and A just starts using Bs driveway to reach the public road. Is this a public license? (1) Unaccompanied by any other factors is likely to be insufficient. (2) The claimant must prove that his reliance is reasonable. (a) Very difficult to satisfy this requirement if the owner of the land had no idea that he was relying on it. (b) If A fails to object and B claims later that easement is created by estoppel, well have a hard time to show the reliance by B is reasonable. c. Lasts only so long to allow the holder of an easement by estoppel to accrue the investment he has made. 2. Minority Rule: a. License can be revoked by the licensor regardless of detrimental reliance by licensee, i.e., STs in the minority do not recognize the doctrine of easement by estoppel 3. Typical Situations a. Grantor gives grantee permission to use his property, which is reasonably construed by the grantee as irrevocable permanently or for a fixed period of time b. Grantor intends to grant an easement (irrevocable) but fails to satisfy Statute of Frauds c. Grantor conveyed an easement over land he did not own, but acquires the land later d. Owner intends to create a revocable license but fails to assert his revocation right (or otherwise make it clear that the license is revocable at will), while owner knows (or should have known) that the grantee mistakenly believes that the license is irrevocable i. By acquiescence, owner implicitly promised that license would extend to allow the grantees intended use. License is converted into a perm (or semi-perm) easement ii. See Holbrook v. Taylor (p. 677) (A) Facts: (1) s built a residence on prop adjoining landowners' prop (2) With landowners permission, the homeowners used and maintained an access road owned by the landowners during home construction. (3) After the construction, the homeowners continued to use the roadway to access the public highway. (B) Holding: evidence justified the finding of the trial court that the right to the use of the roadway had been established by estoppel. (1) Use of the roadway by the homeowners to get to their home from the public highway, the use of the roadway for the construction of the residence, the general improvement of the premises, and the maintenance of the roadway, all with the actual consent of the landowners or at least with their tacit approval, clearly established that the license to use the subject roadway could not have been revoked (C) Was a license created?

63

D.

(1) Yes (D) Reasonable reliance that the license wouldnt be revocable? (1) Yes (E) Did the owners know that the Taylors relied on the irrevocability of this license? (1) Yes-there was no other easy access to the public road. The Taylors were building a family residence. They were depending on the constant usage of this driveway. Easement by implication 1. Easement implied from prior use a. Circumstances giving rise to such easements: i. Servient and dominant estates were owned by the same person prior to division (A) O owns 2 lots, As residence is built on lot 2. A uses lot 1 to access the public road. (1) O transfers lot 1 to A. however, the deed doesnt mention any easement held by O against the property now owned by A. to cover this scenario, the law says that they will grant an easement based on prior usage. (2) All the elements would help the court establish the intent of the parties. ii. One portion of the property was used in an easement-like fashion for the benefit of another part of the property (A) Quasi-servient estate burdened part of the prop (B) Quasi-dominant estate benefitted part of the prop iii. Then, a severance occurred, so that the 2 parts are owned by 2 different people iv. Issue: does the separation implicitly create an easement, so that the quasi-dominant estate can continue benefitting from the quasi-servient estate b. Required elements i. Common ownership prior to severance, but separate ownership after severance (A) There is no easement while one person still owns both props (B) One common owner owned 2 parcels of land in the beginning. ii. Existing Use at Division of the Property (A) Pre-existing use upon the severance of the 2 parcels of land. iii. Parties Intended for the Easement to Continue at the Time of Severance (A) Factors to Construe Intent (1)Reasonable necessity for easements existence (a) Is easement important to the enjoyment of the land? (b) Reasonable necessity creates rebuttable presumption that parties intended to continue easement at time of severance i. If the drive way located on lot 1 was the only way to access the public road, then presumably the parties intended this easement to continue because otherwise it would be difficult for lot 2 owner to get out of his property. ii. Only requires merely REASONABLE necessity. iii. If the common owner claims an easement by prior use that benefits himself against the land that he has transferred out, the degree of necessity required by law is higher.

64

(c) **Strict necessity means that the property is essentially useless w/o the easement, e.g., land-locked prop (2)Price of the property (3)Continuous prior use of quasi-servient estate (a) Continuous not constant (4)Whether prior use was apparent to the parties at severance (a) Prior use is apparent if it can be detected or inferred upon reasonable inspection of the property. Actual visibility is not required (Example: Van Sandt v. Royster, page 682) (5)Higher degree of certainty in proving parties intent to continue prior use of easement in cases of implied reservation or grant (a) Majority parties intent to continue easement must be est. w. greater certainty (b) Minority if grantor maintains quasi-dominant estate, then the grantor must explicitly list the easement in the conveyance or else there must be strict necessity for its continuance (B) Example: Van Sandt v. Royster, page 682 (1) Facts: (a) knew his house was connected to a sewer when he purchased it but was unaware of the location of a lateral sewer drain (b) Lateral sewer drain ran through his property and had been constructed 30 years earlier for the use of neighboring properties as well as his own. (c) There was no mention of an easement in any deed. (d) Case arose when sewage backed up into his basement. (2) Held: (a) When an owner utilized part of his land for the benefit of another part, a quasi easement existed, AND if the owner conveyed the quasi-dominant estate, an easement corresponding to the quasi easement usually vested in the grantee, provided that it was of an apparent continuous and necessary character. (b) Parties were assumed to intend the continuance of uses that were in a considerable degree necessary to the continued usefulness of the land, especially those necessary uses that had so altered the premises as to make them apparent upon reasonably prudent investigation. (c) The existence of plumbing fixtures and lines in 's house made the easement apparent although it was not visible (3) Applied (a) Continuous b/c s regularly used sewer drain (b) Necessary b/c easement was important to s enjoyment of land (c) Apparent due to plumbing fixtures/lines in s house c. Implied Reservation or Grant i. Reservation: grantor (common-owner) keeps the quasi-dominant estate, impliedly reserving in him an easement by continuation of prior use

65

ii. Grant: grantor (common-owner) keeps the quasi-servient estate, giving the grantee an implied grant of easement 2. Easement implied from necessity a. General Rule i. Easements are implied by necessity when an owner sells a landlocked parcel ii. Buyer has a right of access thru grantors remaining land, to the public road b. Issue: is easement by necessity a mandatory rule, or result of constructing parties intent? i. Some courts view it as a result of constructing parties intent (A) If parties clearly did not intend to create easement at time of transfer, then a subsequent land-locking does not give rise to easement by necessity (B) Subsequently landlocked owner should negotiate w. neighbors for easement ii. Some courts: right to access ones own land is given (and protected) by law c. Why do we have this? i. Promote more efficient use of the land? ii. Look at the intent of the parties? (A) At severance, if there is no other way for lots 2 to access the public road, the parties must have intended to have some easement in favor of lot 2. (B) Although the deed is silent, it must be because of oversight of the parties. (C) All you have to show is that the parcel is landlocked. (1) However, if the intent of the parties was not the dominant rationale, then we have to prove several other factors that tend to suggest the intent of the parties. d. Required Elements i. Common owner of both lots prior to severance ii. (Some STs) the necessity existed at the time of the severance of the common ownership (as proof of parties intention to keep the easement) (A) Because it existed at the time of the severance, it must be that the parties intended for an implicit easement to be established. (B) It is hard to establish that an easement was implicit when the necessity didnt exist at the severance of the parcels. iii. The servient estate can only be the land-locking parcel (A) Example: A owned 100 acre of land. The east and west ends of the land are public high way. A first sold the 35 acres on the east side to B, and then he sold 35 acres on the west end to C, and the last 30 acres in the middle to D. The conveyance that locked Ds parcel was the conveyance to C. Thus, D had an easement across Cs property, but not Bs property. (B) BUT, which parcel caused the landlock? (1) Example, N. 2, p. 695: O owned lots # 1-5. In numerically ascending order, O conveyed each lot to A. A died intestate. As kids (B F) recd the lots in ascending order (B 1; C 2; D 3; E 4; F 5). F then sues B E, in order to est. easement by necessity. What result? (2) Answer: (a) When A bought lot 4 from O, an easement by necessity was established over lot 4 for the benefit of lot 5 retained by O.

66

(b) When A purchased lot 5, the easement disappeared. (c) When A died intestate and the court distributed the lots, a new easement by necessity was created for the benefit of lot 5. (d) We dont know which of other kid was holding the landlocking parcel b/c 5 distributions happened at the same time. (e) Thus, the solution is to locate the lot with the least damage, and assess the damage equally against all lot owners (f) As a result, everyone then suffers an equal amount of loss iv. Note: Prior use is not required in order to an easement by necessity e. What if someone is landlocked after the severance and there is no easement granted/implicit? i. The landowner can go ask for an easement from the other people to cross their property. ii. Under the common law, if the other landowner doesnt agree to the easement, then he is SOL. f. Degree of necessity: i. Majority: Require strict necessity (A) Even if available access is by foot down a steep cliff and building a new road would cost $700K, strict necessity does not exist (B) Some courts deny strict necessity when only access is by water ii. Minority: Necessity exists when the alt. access is inadequate, difficult or costly. g. Duration of the easement: i. For as long as the necessity exists. ii. Note: Contrast easement implied from prior use cases, where necessity was used as circumstantial evidence on parties intent. Cessation of necessity does not terminate easement. h. Location of easement implied from necessity: i. Owner of servient estate chooses location for easement implied from necessity because he knows where the easement can be satisfied at the lowest cost. i. Example: Othen v. Rosier (p. 689) i. Facts: (A) landowner lived on prop that was a land-locked parcel (B) For several years, used a roadway on () burdened landowner's prop (C) s subsequently erected a levee, which affected the quality of roadway ii. Arguments: (A) argued that he had an easement of both necessity and by prescription (B) burdened landowners maintained that no easement existed iii. HELD: landowner did not show an implied easement (under deed that he recd) b/c record did not show that the roadway was a necessity as of the date of the deed iv. MINORITY RULE: Privity of ownership is reqd for easement by necessity v. MAJORITY DOES NOT REQ. PRIVITY FOR EASEMENT BY NECESSITY Easement by prescription

E.

67

1. General Rule a. Similar to adverse possession, if claimant has been using the right of way throughout the SOL period, then easement by prescription is established i. Adverse possession gives you the possessory right while easement by prescription just gives you the right to use the property. b. Exceptions: i. Not applicable against Fed or ST govt ii. Not applicable for trees hanging over neighbors property c. Prescriptive Easement or Adverse Possession i. Prescriptive easement: party seeks guarantee for use, not exclusive possessory right (A) Whether the TO is excluded from use is an important factor 2. Elements of Prescriptive Easement a. Overview i. Actual Use, ii. Adverse use and under claim of right, iii. Open and notorious, AND iv. Continuous Use, & v. (In minority jurisdictions) Exclusive use b. Actual Use i. Requires actual, physical presence on the servient estate ii. Negative easements: (A) Negative easements do not satisfy actual use req. (1) There hasnt been a cause of action, thus not tolling the statute of limitations. (B) E.G. A has a house with ocean view. B owns vacant lot b/w As house and the ocean. 11 yrs later, B builds another house on vacant lot that completely blocks As view. Can A claim negative easement by prescription? (1) Answer: no, b/c A lacks actual, physical presence on Bs estate c. Adverse Use and under claim of right i. Some presumptions (A) Non-permissive use is presumed from mere acquiescence by landowner (B) But, if claimant of easement is immediate family member of landowner, then permissive use is presumed from mere acquiescence by landowner ii. A person who enters pursuant to a defective deed enters by claim of right, not by permission. His use is hostile and adverse iii. Adverse nature can be terminated by permission from landowner (A) If A and B owns adjacent land, and B builds a mailbox on As land. One year before the statute of limitations runs out, A discovers the mailbox and allows B to have it there, adverse use fails because he has permission now. (B) But, if the use exceeds the scope of permission, then the unauthorized use can ripen into easement iv. Continued use after termination of a prior easement is adverse (A) Example: A and B own adjacent lots. A acquired an easement by necessity 5 years ago from B since A could only access the public road through Bs land. The city has just opened up a new road to which As land is connected. As easement by necessity ends. (1) If A continues using the easement for SOL period and satisfies all other prescriptive easement reqs., A has an easement by prescription

68

v. Claim of Right (A) Objective view (i.e., mental state of claimant is irrelevant). (1) Most jurisdictions go by this standard. (B) Subjective Aggressive (C) Subjective Good-Faith d. Open and notorious use i. The use must be such that it may be discovered by reasonable inspection ii. Open & notorious is not reqd. if TO has actual notice of use e. Continuous use i. Constant use is not reqd. (A) Seasonal use is sufficient, if it is typical of similarly situated land ii. An interruption of use resets the SOL period (A) But if A tries to interrupt and B just ignores it, then the interruption doesnt count and the statute keeps tolling. iii. What constitutes interruption? (A) TOs action must be effective in stopping the adverse use (1) A request to stop adverse use, if unheeded by the adverse user, will usually be insufficient to interrupt the continuous use (2) Adverse use must be effectively stopped by TOs action (B) If the adverse user resumes the use in just a short period of time, the interruption is unsuccessful (1) How short is short? 1 day, 1 week, 1 month? 1 year? (a) Answer: There is no fixed rule. It must be examined in light of other facts to see whether there was an intention to abandon the use when the use was first stopped (C) Standard for interruption is lower in public areas, e.g., a symbolic interruption is good enough. (Temp. sign: Not Open to Public) f. Exclusive use: i. Most jurisdictions do not require exclusive use (A) In prescriptive easement cases, exclusiveness does not mean the claimant is the only user. The prescriptive user needs not prohibit others from using the property, so long as he prevents interferences with his use. (B) There are cases in which exclusivity was satisfied even though the easement was also used by the servient estate owner. ii. Minority: usage of easement in common w. other people is not exclusive. g. Prescription period: usually, SOL for prescriptive easement and adverse possession is same 3. Note: negative easement cannot be created by prescription because the servient estate owner has no cause of action against the claimed negative easement during the limitation period. IV. Transfer and division of easement A. Transfer of easement appurtenant Appurtenant easement benefits the dominant estate and thus it binds the subsequent owners of the servient estate and dominant estate. B. Transfer of easement in gross 1. Early cases: Not transferable. a. The presumption is that easement in gross is not transferable unless the parties intended otherwise. 2. Recent cases: Most cases hold that easement in gross is transferable if that is intended by the parties. Recreational easement that are typically personal in nature (e.g., fishing, hunting, camping) are presumably not transferable unless there is

69

V.

evidence of the contrary intent of the parties. A commercial easement in gross is transferable. C. License: 1. Binds only the licensor and the licensee, unless otherwise intended by the parties. Upon transfer by either party, the license ends. D. Division of easement 1. Appurtenant easement - division of dominant estate Division of dominant estate often results in increased burden on the servient estate. The general rule is that the dominant estate may be divided and that each part of the divided estate is entitled to enjoy the easement if the increased burden on the servient estate is within the original contemplation of the parties. In constructing intent of original easement parties, courts have considered evidence such as whether the division of the dominant estate is a normal development, whether the division is foreseeable, and whether the burden on the servient estate is substantially increased. 2. Easement in gross Again, intention of the parties is key. If the holder of the easement is the sole beneficiary of the easement, courts presume that the parties have intended for the easement to be divisible at the option of the easement holder because he is the only person who will bear the consequence. Of course, this presumption can be rebutted by contrary evidence. Exclusive easement in gross right to B to go over the pond to fish-in this situation, the presumption can subdivide-he is the only person who enjoys the pond. If he wants to share it with someone else, he can do it. Scope of easement A. Easement created by express grant 1. General rule: the language of the express easement defines the scope of the easement. When the grant is explicit as to the scope of the permitted use, neither party can unilaterally change that. 2. When the express grant is silent about the use requested (example: Preseault v. United States, page 725) In such situations, most courts try to infer the intent of the parties. Key factors that influence courts decision include: a. Is the requested use similar in nature as those explicitly granted in the easement? If yes, the omission is likely due to oversight. b. Whether there is substantial increase in the burden on the servient estate. If yes, it is unlikely that the parties had intended to include the requested use in the easement. c. Whether the requested use is reasonably foreseeable by the parties at the establishment of the easement. If no, it is unlikely that courts will find that the parties had intended something they could not foresee. d. If the requested use is a result of the normal development of the dominant estate, it is presumably included in the scope of the easement because courts presume that the parties intended the scope to change in order to accommodate the normal development of the dominant estate. 3. The enlarged dominant estate scenario a. General rule: The dominant estate cannot be enlarged geographically so as to include another land that was not subject to the original easement, even though such inclusion does not unreasonably increase the burden of the servient estate. i. What if we cant tell the difference between the parcels anymore? (A) Should injunction be granted against the party? (B) If you cant distinguish the land, segregation has to stop. b. Injunction not always granted: Courts have sometimes refused to grant injunction when the burden on the servient estate is not substantially increased.

70

Instead, courts grant monetary damage to the servient estate (example: Brown v. Voss, page 716) i. Even though the inclusion of another parcel is not substantial, in an explicit easement, the party doesnt have the legal right to include the added parcel in the easement. 4. Scope of easement by implication a. Scope of easement implied from prior use i. In 1990, A owned B. Easement implied by necessity Scope is dictated by exact necessity: no more, no less. C. Easement created by prescription Scope is construed narrowly. Although scope is not confined to actual use, courts have been reluctant to expand original scope to accommodate changed future needs of the dominate estate. The claimed use must be of similar kind as the adverse use and must be what the servient owner might reasonably expect to lose by failing to interrupt the adverse use. Courts presume that the servient estate owner who did not sue when the original adverse use occurred would have objected to a kind of use that was substantially different (example: A acquired prescription easement over Bs lot to access the pubic road on foot. Can A ride a bike on this easement road? Answer: depends on specific circumstances, but it can be argued that it is of similar kind as the adverse use. What about motorcycles? Answer: likely NO). D. Additional notes on scope of easement 1. Use by servient estate: Servient estate is allowed to use the easement unless the easement has granted an exclusive right to use to the easement holder. The servient estate may use the easement to the extent that the use is reasonable and does not substantially interfere with the easement holders use. 2. Overlapping easement: Servient estate may also grant to third parties overlapping easements if the easement held by the first dominant estate is not exclusive, and the grant to third parties does not unreasonably interfere with the first dominant estates right to use. 3. Change in location of easement: The established rule on this issue is that once fixed, the location of an easement is permanent unless the parties agree to a change. However, the Restatement of Property (representing the growing trend) has changed this rule. It grants the servient owner the right to change the location of an easement if the change does not significantly lessen the utility of the easement, increase the burdens on the holder of the easement in its use and enjoyment, or frustrate the purpose for which the easement was created. VI. Termination of easement Ways to terminate an easement include: expiration, merger, cessation of necessity, abandonment by easement holder, prescription by the servient estate owner, change or destruction of the dominate estate in such a way that the easement may no longer be used. A. Expiration If the easement grant has set a time limit for the existence of the easement, when that time limit comes, the easement is terminated automatically. If the original grant saw that B holds an easement over As land for 10 years. Keep in mind, if B continuously uses this easement even after termination, he may end up acquiring this easement again if he can satisfy the elements of adverse possession (easement terminology) Merger When the servient estate and the dominant estate are owned by the same person, the easement is terminated because easement applies only to right to use someone elses land (example: in Brown v. Voss, page 716, Brown was holding parcel B which

B.

71

had an easement over Voss parcel A. When Voss subsequently purchased parcel B, the easement was terminated). No one can hold an easement over his own property. If A owns lot 2, B owns lot 1. Later on, A acquires lot 1 from B, and alternatively B acquires lot 2 from A, the ownership between the serviant and dominant estate merge and eliminate the existing easement. C. Cessation of necessity For easements implied by necessity, when necessity stops, the easement is terminated. But you may ask, we thought that we granted the easmenet to interpret the partys intent. If they intended the easement to be created, why does the cessation of the necessity get rid of the easement? Abandonment by the easement holder An easement may be terminated by abandonment if the easement holder shows a clear and unequivocal intent to abandon the easement. Whether the easement holder intended to abandon is often controversial. B has lost his right to use the easement right away. What constitutes abandonment? No fixed rule, but we need to consider whether B has shown an unequivocal intent to abandon this easement. In this regard, mere non-usage no matter for how long is not sufficient to constitute abandonment. 1. Nonuse is not abandonment Mere nonuse by the easement holder, no matter for how long, is not enough to constitute abandonment. Some action on the part of the easement holder, or failure to assert his right when it is violated, is required in addition to nonuse (example: Preseault v. United States, page 725).

D.

C. Easement by prescription can be terminated by the true owners own prescription. If B has
been using As driveway for 10 years, and this acquires prescriptive easement, A upset by this fact starts putting a train at the entrance of this easement which is effective in barring Bs use of the driveway. As conduct lasts throughout another statute of limitation periodthis terminates the original easement obtained by B through prescription. Exclusive use to the extent that it prevents the original easement owner by using the property.

Co-ownership and Marital Interests


I. Scope of coverage A. Different types of co-ownership: 1. Tenancy in common, 2. Joint tenancy, and 3. Joint tenancy with right of survivorship B. Partition of property C. Sharing the benefits and burdens of the property D. Common law marital interests Tenancy in common A. Definition 1. Concurrent ownership

II.

72

B.

2. Multiple persons own the same property at the same time 3. Each tenant in common owns the right to possess the entire property a) Each tenant can go into each others apartment. Characteristics 1. Uneven shares and different estates are permitted a) E.G. A, B, C, and D own land in equal shares as tenants in common. A conveys his interest to B. B, C, and D are still tenants in common, but B owns interest, while C and D each own interest b) Note: there is a rebuttable presumption that tenants in common have equal shares in the property 2. Transferable by deed, devisable by will, or inheritable by intestacy a) Freely transferable. b) If A and B hold a tenancy in common and A transfers his interest to C, then B and C are tenants in common. 3. No survivorship interest among co-tenants a) Upon the death of one of the tenants, his interest belongs to his estate or the person to whom he has transferred by will. 4. Right to possession of entire property by each tenant in common

III. Joint tenants A. B. Definition 1. Similar to tenancy in common, BUT there is right of survivorship among joint tenants Characteristics 1. Survivorship in living joint tenant a) Simultaneous death of A and B: each joint tenant receives interest in property b) Murder by one joint tenant: murderer loses right of survivorship c) Cannot be devised by will or inherited: tenants interest terminates upon death. d) If A dies a second before B, then B gets 100% of the estate. 2. Joint tenants are deemed one person in holding interest in this property. a) Because they are deemed one person, naturally there are certain unities that exist for their joint ownership interest. 3. 4 Unities a) TIME: i. Interest of each joint tenant must be acquired or vest at the same time ii. E.G.: O conveys land to A and B as joint tenants on the dates each is admitted to the Ohio bar. If A and B are not admitted to the bar on the same date, they are not joint tenants because unity in time is missing. b) TITLE: i. All joint tenants must have acquired title by the same instrument or by a joint adverse possession. ii. A holds a fee simple in his house and he wants to give B a gift by conveying the house to A and B as joint tenants. (1) Right now in this conveyance, they arent joint tenants because other unities are missing. (2) Unity in title is missing because they both acquired the land in different deeds. (A) It needs to be on the same piece of paper. c) INTEREST:

73

i. ii.

iii.

All joint tenants have equal undivided shares and identical interests measured by duration E.G.: O conveys To A and her heirs, and to B and his heirs so long as B is married to A, as joint tenants, otherwise Bs interest goes to C. (1) A and B hold the property as tenants in common and NOT joint tenants because the durations of their interests are not the same. (2) A holds a fee simple absolute, B holds a fee simple determinable. MODERN: courts are increasingly ignoring this requirement. Instead, there is only a rebuttable presumption of equal interest.

d) RIGHT of POSSESSION:
When created, each joint tenant must have a right to possession of the whole (1) After the creation of the interest, one of the joint tenants can by agreement give exclusive possession to the other tenant (2) It constitutes a voluntary waiver of the right to possess the whole (3) Such agreement is enforced without destroying the joint tenancy, due to absence of unity of possession 4. Destruction of joint tenancy a) Destruction of 4 unities i. Traditional Common law: (1) All 4 Unities must be preserved (2) If any unity is severed, then it converts into a tenancy in common ii. Modern view: (1) If evidence shows that the parties did not intend to sever the joint tenancy, joint tenancy will exist even though 4 Unities are absent. b) Destruction by mutual agreement: i. Joint tenancy can be destroyed by agreement among joint tenants. c) Destruction by unilateral transfer: i. Destroyed if one joint tenant unilaterally conveys his interest to a 3rd P ii. This severs the joint tenancy as b/w this 3rd P and the other joint tenants b/c it destroys at least the unity of time of their interests. iii. 3rd P can be a strawman, who returns interest to conveying tenant iv. Other joint tenants need not be aware of the conveyance at the time it occurs, but some STs require that the conveying joint tenant record the transfer w/i a reasonable or statutory time in order to avoid surprise transfers v. Joint tenancy is severed only with regard to the interest conveyed vi. E.G.: A, B and C held the land as joint tenants. A subsequently conveyed his interest to B. Suppose at Bs death, he left in his will all his interest in the property to his son S (1) What is the extent of Ss interest? (2) Answer: (A) S holds 1/3 of the interest as tenants in common with C. (B) C holds the other 2/3 of the interest by himself. (C) Bs joint tenancy in that 2/3 of the interest is not devisable and upon his death, Bs interest goes to C. i.

74

vii. E.G. A, B, and C hold property as joint tenants. Then, A conveys his interest to D. (1) As transfer to D severs the joint tenancy to 1/3 of the interest. What about between B and C? Is the joint tenancy between them also destroyed? NO. (2) Then, B dies w/o a will, leaving H as his heir (A) What is the state of the title? a. D holds 1/3 interest as tenant in common b. C holds 2/3 interest as tenant in common d) Destruction by a joint tenant conveying his interest to himself i. See Riddle v. Harmon (p. 280) (1) Wife conveyed interest in joint tenancy to herself (2) HELD: common sense as well as legal efficiency dictate that a joint tenant should be able to accomplish directly what he could otherwise achieve indirectly by use of elaborate legal fictions ii. Modern Rule: a strawman is not reqd. to terminate a joint tenancy iii. BAI: how could one protect the survivorship interest? (1) Create a joint life estate interest, remainder going to survivor (2) E.g. to A and B for life as joint tenants and not as tenants in common, remainder to survivor of A and B iv. Other joint tenants need not be aware of the conveyance at the time it occurs, but some STs require that the conveying joint tenant record the transfer w/i a reasonable or statutory time in order to avoid surprise transfers (1) A conveys on a piece of paper that she hides her land to herself, but hides it just in case B dies before her. (2) In order for the conveyance that destroys the joint tenancy to be valid, it needs to be recorded within a reasonable amount of time in accordance with the state statutes. e) Taking out a mortgage on the property: i. Title theory jurisdictions (1) i.e. mortgage passes legal title to lender (2) Taking a mortgage severs joint tenancy. ii. Lien theory jurisdictions (1) i.e. mortgage merely gives lender a lien on prop, not a title to prop. Title passes only after the foreclosure of a mortgage and upon delivery of the masters deed under the foreclosure sale (2) Taking a mortgage does not sever joint tenancy (3) See Harms v. Sprague, p. 285 (following Lien Theory) iii. NOTE: (1) The mortgage lien taken out by one joint tenant on the joint property does not survive the death of the joint-tenant mortgagor iv. Harms v. Sprague (1) Facts: (A) John and William Harms held property X as joint tenants (B) Sprague wanted to purchase property Y, but needed money (C) John Harms placed mortgaged his joint interest to gain money (D) John co-signed w. Sprague and provided the needed money

75

(E)

Then, John Harms died devising his interest in X to Sprague

(2) Issue: (A) Does the mortgage by John Harms on property X destroy the joint tenancy b/w John and William Harms? (3) Held: (A) Applying the Lien Theory of mortgages, the joint tenancy was not destroyed by John Harms actions (B) Thus, John Harms interest in the prop terminated when he died a. One cannot devise a joint tenancy interest (C) Furthermore, John Harms mortgage terminated when he died. (D) Therefore, William Harms owns property X free and clear (4) How can you protect yourself if you are the creditor if you know the mortgage will not survive? (A) All joint tenants signing on this mortgage. (B) Taking out life insurance on the debtor (C) Have the debtor convey his interest into a 3rd person to sever the joint tenancy first. (5) BAI: (A) If William Harms had died first and John Harms had defaulted on his mortgage, then the lender could repo the entire property f) Lease out the property by one joint tenant i. Common law: (1) The lease severs the joint tenancy because the unity of interest was destroyed (A holds only a reversion while B holds fee simple). ii. Modern view (majority jurisdictions): (1) Leasing out the prop by one joint tenant does not sever joint tenancy. iii. Note: (1) Lease loses binding effect when its creating joint tenant dies (2) Holding otherwise would lead to the result that a joint tenant can bind not only his interest but also the survivorship interest of his joint tenant. iv. A and B are joint tenants for an apartment. B leases out his apartment to C for 10 years. The second day after the lease, B dies. Does the lease survive the death of B? (1) Does the property go back to A free and clear right after Bs death? (A) The lease does not survive the death of B. A takes the property back free and clear of the lease between B and C. (B) What is Cs protection? a. Do a good title search before signing a lease. Find out if this piece of property has joint tenants. If it is, C needs to make sure that all joint tenants sign the lease. g) Divorce of husband and wife joint tenants i. Majority jurisdictions: Divorce alone does not sever joint tenancy. (1) Explicit agreement b/w husband and wife to sever joint tenancy is reqd.

76

(2) Provisions in divorce agreement re allocating properties upon divorce do not constitute an explicit agreement to sever the joint tenancy. ii. Minority jurisdictions: Some STs have statutes providing that divorce converts a joint tenancy, b/w former spouses, into tenancy in common. iii. H and W, owners of Blackacre as joint tenants, are getting a divorce. They sign a divorce agreement providing that Blackacre will be sold and the proceeds divided equally between H and W. Before Blackacre is sold, W dies. Does H have survivorship rights in Blackacre? (1) The provision is not explicit enough of their intent to sever the joint tenancy. In order to sever the tenancy, it needs to say we agree to sever the joint tenancy.. 5. Joint tenancy in bank accounts a) Survivorship i. Majority jurisdiction: Surviving joint tenant takes remaining deposited money unless there is clear and convincing evidence that survivors name was on the account only with the intent to create a convenience account. (1) The burden of proof is on the person challenging the survivorship ii. Minority jurisdiction: To avoid litigation, the presumption of survivorship is conclusive, regardless of any evidence to the contrary. iii. **Contents of a bank account are treated the same as a bank account b) The present rights of each joint tenant: i. Majority jurisdictions: Joint tenants of bank accounts are presumed to own the amount in proportion to his net contribution. This presumption can be rebutted by evidence of intention of gift from one joint account holder to the other. ii. Minority jurisdictions: Each joint tenant holds equal fractional interest 6. Creating joint tenancy a) Language used: To A and B as joint tenants and not as tenants in common. i. Some STs req. phrase with right of survivorship to create a joint tenancy. (1) In such a state, the language to A and B as joint tenants and not as tenants in common is not good enough to create a joint tenancy. b) Presumptions in case of ambiguity: i. Modern View prefers tenancy in common over joint tenancy (1) E.G.: To A and B jointly. (A) This is a vague grant, tenancy in common is created. (B) Most states will regard a tenancy in common is created. (C) BUT, if there is a provision for survivorship, it is a joint tenancy (D) Example: note 4, page 278, the deed to grantees was jointly, as tenants in common, with equal rights and interest in said land, and to the survivor thereof, in fee simpleThe court held that a joint tenancy was created because it provided for survivorship). IV. Tenancy by the entirety A. Creation of tenancy by the entirety

77

B.

tenancy by the entirety, but this doctrine applied to real property, not to personal property a) To H and W are joint tenants, a tenancy by the entirety is created. i. All the characteristics of a joint tenancy with an addition of the unity of marriage. b) A and B were about to get married. Prior to the wedding ceremony, A and B went out to purchase their future home. They signed the deed as tenants by the entirety. They got this house 10 years later. B transfers his interest to C, then dies. Does A get the property? i. Neither party/co-tenant can unilaterally destroy the survivorship interest of the other co-tenant. Neither co-tenant can unilaterally seek the partition of this piece of property held in the form of tenancy by its entirety. ii. No because tenancy by the entirety can only exist as between husband and wife. When A and B went out, they intended to get married soon, but they werent yet husband and wife. (1) Therefore either a joint tenancy or tenancy in common. Therefore, Bs transfer to C severs survivorship interest. 2. Modern day: 1/2 US jurisdictions have abolished tenancy by the entirety. a) In jurisdictions that still have tenancy by entirety, a tenancy by the entirety is presumed when a gift is made to both husband and wife b) This presumption can be rebutted by evidence to the contrary c) In these jurisdictions, tenancy by the entirety applies to personal and real property i. But, not applicable to bank deposits b/c its hard to ascertain if survivorship interest is violated when both spouses have unlimited access to bank deposits. Characteristics 1. 5 unities (i.e., 4 unities of joint tenancy + the 5th unity of marriage). a) Note: tenancy by the entirety can only exists b/w spouses 2. Destruction of Tenancy by the Entirety a) No unilateral act can destroy tenancy by the entirety, or attached survivorship interest b) Neither husband nor wife can unilaterally seek judicial partition of the property. c) Divorce can terminate tenancy by the entirety.

1. Traditional common law: a conveyance to husband and wife necessarily creates a

V.

Partition of property among co-owners A. Partition by agreement honored 1. Joint tenants or tenants-in-common can privately agree to divide up the common property 2. When such agreement cannot be reached, the tenants can resort to judicial intervention B. Judicial partition 1. Partition in kind v. partition by sale a) Intro i. Courts favor partition in kind (1) Property will be kept together (2) Emotional attachment to the land ii. A sale of ones property w/o his consent is an extreme exertion of judicial power warranted only in clear cases b) Partition in kind: i. Division of the property itself among the co-owners.

78

C.

Constitutes a forced sale of the land, followed by division of the profits thus realized among the tenants. d) When is partition by sale ordered? i. In-kind partition is impracticable due to the physical attributes of the land ii. AND, interests of all co-owners are better promoted by a partition by sale iii. **Party seeking sale is burdened to establish the 2 elements above e) What factors determine whether interests of all parties are promoted? i. Economic values recd by parties from in-kind partition v. partition by sale (1) Example: Johnson v. Hendrickson, Note 2, page 298 (A) Facts: in 1904, the owner of 160 acres of farm died, conveying 1/3 of land to his widow and 2/3 to each of his 3 children. Widow remarried and had twin sons by her second husband. The deceaseds children grew up and left home. 2nd husband and twin sons purchased a home across the street from original property. When mother died leaving her interest to twin sons, the children from first husband sued to force partition by sale. (B) Holding: partition by sale granted a. Land is owned by 6 people b. Dividing the farm into 6 pieces would undermine its value, re both its salability and its agricultural uses (2) Proof of substantial economic loss is relevant to determination, but evidence of forfeiture of benefits is probably not ii. Existing, strong emotional attachment (e.g., family dwelling or business) to the property can outweigh economic justification and lead to partition-in-kind (1) Example: Arkland Co. v. Harper, Note 2 page 298 (2) Example: Delfino v. Vealencis, page 292 (A) Facts: and owned 20.5 acres as tenants-in-common. owned about 69%, while owned about 31%. occupied the dwelling and operated a garbage removal business. wanted to develop the property for residential housing. proposed a partition by sale, w. proceeds being paid proportionally. (B) Holding: partition in kind granted a. Not only would lose her business, but also would be booted from her home. b. These factors outweighed the s concerns that the residential properties would be sold for less money and a residential road would need to be re-routed around iii. Feasibility of partition in kind iv. Incapability of use. v. Interest of harmonious co-occupancy The effect of agreement not to partition 1. Agreement may be invalidated as undue restriction on alienation unless it is for a limited period of time that is reasonable for achieving a legitimate purpose of the restriction.

c) Partition by sale:
i.

79

VI. Sharing the benefits and burdens of the property A. Obligation by occupying tenant to pay rent to cotenants 1. Majority view: No obligation to pay rent in the absence of ouster a) The person has the right to occupy the entire property in the first place. 2. Minority view: The occupying tenant must pay rent to cotenants even in absence of ouster. 3. What constitute an ouster? a) Non-occupying tenant is denied entry to the property b) Refusal to pay rent or vacate property upon demand is not enough for ouster i. Example: Spiller v. Mackereth, page 300 c) A and B occupy a piece of property as storage. B moves out and C comes in. A wants C to pay rent. C puts a new lock on the door. Is this an ouster? NO. i. What facts need to be added? (1) A asks C for a key and C refuses. A would be entitled to his share of the fair rental value of the entire property. ii. What if after B moves out, C wants to lease the entire property to D over As objection. B. The right to lease out property without consent of cotenants 1. Generally a) Co-tenant can lease out the entire prop w/o other co-tenants consent b) The non-consenting co-tenant has no right to expel the lessee c) The lessee is not liable to anyone for rent except to his lessor d) But, if lessee refuses to allow non-consenting co-tenant to enter prop, then an ouster has occurred which entitles non-consenting co-tenant to receive proportional rent i. Example: Swartzbaugh v. Sampson, page 303 2. The non-consenting co-tenants remedy a) Seek partition at court b) To enter and be ousted. Now, remedies for ousted co-tenants are available: i. Proportional share of reasonable value of leased props use and occupation ii. Seek accounting for benefits from the leasing co-tenant C. Accounting for benefits and contributing to costs 1. Right to accounting: a) In all states, a co-tenant who collects from 3rd parties rents and other payments (e.g., selling timber on the land) must account to other co-tenants for the net amounts received (net of cost) in excess of his proportional shares. b) In absence of ouster, accounting is based on actual receipts, not fair market value. 2. Contribution for carrying charges (tax, mortgage, etc): a) A co-tenant paying more than his share of taxes, mortgage payments, and other necessary carrying charges has a right to contribution from the other co-tenants. b) Exception: the tenant who has paid all such charges has sole possession of the property during the period for which the charges are assessed i. Example: A and B are joint tenants. A has paid all taxes for the current year and has sole possession of the prop. Many STs (not all) require B to contribute to the tax only to the extent that tax > rental value of the property. 3. Necessary repairs: a) In most jurisdictions a co-tenant making or paying for necessary repairs has no affirmative right to contribution from the other co-tenants in absence of an agreement.

80

i. Such expenses are not credited in partition sales. b) Rationale: The law seeks to avoid disputes on whether expenditures are justified. 4. Improvements: a) No right to contribution from other co-tenants for expenditures for improvements. b) But, in partition proceedings, the part of the property that has been improved upon is allocated to the improver, or the proceeds are allocated in such a way that any increase in value due to the improvement is allocated to the improver i. i.e. not reimbursed for costs, but entitled to the benefit of the improvement VII. Marital Interests A. 2 Major Systems 1. Common Law Marital Property 2. Community Property B. Property rights of husband and wife during marriage 1. Traditional common law: The husband has complete control and use of the property held with wife as tenants by the entirety during the marriage. The husband could transfer this right as well as his right of survivorship. Wife could not transfer her right of survivorship in property held as tenants by the entirety without her husbands consent. 2. Woman can have property rights in her name if she is single, but she loses that right when she becomes married. 3. Disposition of the property-the husband has the sole right to transfer or dispose of it entirely. a) Implication for creditors: the husbands creditors can reach all the property held by the couple as tenants by the entirety, subject to the right of survivorship of the wife. The wifes creditors could not reach the entirety property. 4. Modern view: All states have passed The Married Womens Property Acts, which have been interpreted by courts to equalize wives with husbands in terms of property rights. a) Property held in each spouses own name: i. Generally, a married woman is able to receive, hold, manage, and dispose of real and personal property that are in her own name as if unmarried. b) Unilateral Transfer of property held in a tenancy by the entirety: i. The Married Womens Property Acts have not specifically provided for the wifes interest in a tenancy by the entirety, and thus it falls to courts to apply the principle of the Acts to such property. There are different views: ii. Minority view: husband and wife enjoy the right of alienation. But, neither spouse can destroy the right of survivorship of the other. (1) Implications for creditors: creditors of either spouse can levy on his or her interest, subject to the possessory right of property of other spouse and survivorship right to the whole of other spouse. (2) Consequences of levy by creditor of one spouse: The creditor becomes a tenant in common with the non-debtor spouse for the joint lives of the spouses. (A) Moreover, creditor steps into the debtor-spouses shoes and has same rights/obligations under tenants by the entirety.

81

(B) Although he and non-debtor spouse are not tenants by the entirety, neither can unilaterally sever the tenancy by partition. (i.) If the debtor spouse dies first, the non-debtor spouse owns the house free of the creditors interest. (ii.) If the non-debtor spouse dies first, creditor owns the house free of non-debtor spouses interest. (iii.) If spouses divorce, creditor and non-debtor spouse become tenants in common in reality; thus, the creditor and non-debtor spouses survivorship interest ceases to exist, and either party can seek partition. iii. Majority view: (1) Neither spouse has the right to transfer his or her interest without the consent of the other. (2) Implication for creditors: Creditors of one spouse only cannot levy on entirety of property held in tenancy by the entirety iv. Sawada v. Endo, page 313 (exemplifies majority view) (1) s were injured by husband. was uninsured. was holding a real property with his wife as tenants by the entirety. While s lawsuit was pending, and wife conveyed real property to their sons, but and wife continued to live on the property. In 1971, judgment was entered against the defendant husband. After judgment, s wife died and proved unable to pay the judgment. s brought this action to set aside the conveyance to the sons as fraudulent, thereby allowing the s to attach the real property to the suit to satisfy the judgment (2) Issue: whether the interest of one spouse in real property, held in tenancy by the entireties, is reachable by one spouses creditors (3) HELD: injured s, as creditors, had no right to the real property b/c it was held by and wife as tenants by the entirety (4) Ratio: (A) Property held by spouses as tenants by the entirety is indivisible, except by joint action of the spouses (B) Public policy supports protecting the family home, which is the single most important family asset C. Duties of Support the effect of promising property interest in exchange for support: 1. After having a stroke, husband was anois to avoid going into a nursing home and being cared for by professional nurses. Entered into agreement with the wife. Husband promises wife that if she personally cares for him at home, husband would devise her certain property in will. Thus the wife personally cared for him until his death. When he died, husband devised the property to his daughter by a prior marriage. Can the wife enforce the K? 2. See Problem at page 313 3. Answer: Personal performance of a personal duty created by the contract of marriage did not constitute a new consideration supporting the indebtedness alleged by appellant a) Also, the alleged contract was void as against public policy b) Is the wife obligated to provide personal care to the husband? i. Can she hire a professional nurse to take care of her husband?

82

(1) Under traditional common law, the wife is required to provide personal care. But the husband didnt have to give care to the wife. But the husband had to give financial support to the family. (2) But when this was applicable, almost no women worked outside of the family. So there only contribution to the family was to take care of the family. But over the years more woman are in the workplace. D. The rights and obligations upon dissolution of marriage 1. Property rights upon death Dower (SKIP, covered in Wills and Estates) 2. Property Rights upon Divorce a) Is professional degree a marital property? i. Most jurisdictions that have ruled on this issue hold that degrees are not marital property. Contribution from non-degree holder spouse is considered when courts allocate property and award alimony upon divorce ii. In re Marriage of Graham (CO 1978) [p. 322] (1) The court found that the [CO statute that governed divorce] did not explicitly define property to include educational degrees and, as the degree could not be assigned, sold, transferred, conveyed, or pledged, it was an aid which could potentially assist in the future acquisition of property rather than an actual item of property (2) The wife wanted some of the income that came from the husbands MBA because she supported him while he went to school. b) Equitable distribution of marital property: Almost every common law marital property jurisdiction has adopted some form of equitable distribution statute, which gives the court the discretion to divide marital property in a way that the court deems equitable. Problem often arises as to what is a marital property 3. Why Bai rules for wife: a) Marriage is a joined investment in human capital made by both the husband and the wife. i. With this comes with the expectation of return. b) The wife made an assumption of the risk that the marriage might end up in divorce, but she didnt assume the risk that upon divorce her investment wouldnt be compensated. i. She will be deprived of the initial expectation when she made the investment. c) Existing compensation to the wife is not sufficient i. Without being entitled to share the future income stream, the wife can only rely on alimony in this case (1) 2 different types: (A) Prominent alimony (i.) The husband would be responsible for supporting the wife throughout her life (ii.) Rarely granted i. Only when the wife is already at a senior age and unable to make an independent living. (B) Rehabilitative alimony

83

(i.) Granted to wife so she can go out and have her own training to get back in the labor force. (ii.) Highly restrictive i. Will stop upon the occurrence of such events ii. The husband loses his job. iii. Once lost, its lost forever. iv. When the wife remarries. (iii.) If the wife is already working at the time of the divorce, she wouldnt be entitled to this type of alimony. (iv.) Doesnt compensate the wife for the years that have been lost where she could have pursued her own career.

84

Anda mungkin juga menyukai